Paeds Flashcards

1
Q

Gross motor milestones

3 months

A

Little or no head lag on being pulled to sit
Lying on abdomen, good head control
Held sitting, lumbar curve

How well did you know this?
1
Not at all
2
3
4
5
Perfectly
2
Q

Gross motor milestones

6 months

A
Lying on abdomen arms extended
Lying on back, lifts and grasps feet
Pulls self to sitting
Held sitting, back straight
Rolls front to back
How well did you know this?
1
Not at all
2
3
4
5
Perfectly
3
Q

Gross motor milestones

7-8m

A

Sits without support

Refer at 12m

How well did you know this?
1
Not at all
2
3
4
5
Perfectly
4
Q

Gross motor milestones

9 months

A

Pulls to standing

Crawls

How well did you know this?
1
Not at all
2
3
4
5
Perfectly
5
Q

Gross motor milestones

12m

A

Cruises

Walks with one hand held

How well did you know this?
1
Not at all
2
3
4
5
Perfectly
6
Q

Gross motor milestones

13-15m

A

Walks unsupported

Refer at 18m

How well did you know this?
1
Not at all
2
3
4
5
Perfectly
7
Q

Gross motor milestones

2y

A

Runs

Walks upstairs and downstairs holding onto rail

How well did you know this?
1
Not at all
2
3
4
5
Perfectly
8
Q

Pyloric stenosis presentation:

A

Pyloric stenosis typically presents in the second
to fourth weeks of life with vomiting, although
rarely may present later at up to four months. It
is caused by hypertrophy of the circular
muscles of the pylorus

How well did you know this?
1
Not at all
2
3
4
5
Perfectly
9
Q

Features of Py Sten

A

‘projectile’ vomiting, typically 30 minutes after
a feed
constipation and dehydration may also be
present
a palpable mass may be present in the upper
abdomen
hypochloraemic, hypokalaemic alkalosis due to
persistent vomiting

How well did you know this?
1
Not at all
2
3
4
5
Perfectly
10
Q

Dx of py sten

A

USS

How well did you know this?
1
Not at all
2
3
4
5
Perfectly
11
Q

Ramstedt pylorotomy

A

Used in management of py sten
Excision of the hypertrophied circular muscles
of the pylorus

How well did you know this?
1
Not at all
2
3
4
5
Perfectly
12
Q

Def intussuception

A

Intussusception describes the invagination of
one portion of bowel into the lumen of the
adjacent bowel, most commonly around the
ileo-caecal region.
Intussusception usually affects infants between
6-18 months old. Boys are affected twice as
often as girls

How well did you know this?
1
Not at all
2
3
4
5
Perfectly
13
Q

Features of intussuception

A
paroxysmal abdominal colic pain
during paroxysm the infant will
characteristically draw their knees up and turn
pale
vomiting
blood stained stool - 'red-currant jelly'
sausage-shaped mass in the right lower
quadrant
How well did you know this?
1
Not at all
2
3
4
5
Perfectly
14
Q

Ix intussuception

A

uss

How well did you know this?
1
Not at all
2
3
4
5
Perfectly
15
Q

Mx of intussuception

A

Air insuffation under radiological control
If the child has signs of peritonitis or the air
insufflation fails, Sx

How well did you know this?
1
Not at all
2
3
4
5
Perfectly
16
Q

A 2-month-old boy is brought to the afternoon
surgery by his mother. Since the morning he
has been taking reduced feeds and has been
‘not his usual self’. On examination the baby
appears well but has a temperature of 38.7ºC.
What is the most appropriate management?
Advise regarding antipyretics, to see if not
settling
IM benzylpenicillin
Advise regarding antipyretics, booked
appointment for next day
Admit to hospital
Empirical amoxicillin for 7 days

A

Any child less than 3 months old with a
temperature > 38ºC is regarded as a ‘red’
feature in the new NICE guidelines, warranting
urgent referral to a paediatrician. Although
many experienced GPs may choose not to
strictly follow such advice it is important to be
aware of recent guidelines for the exam

How well did you know this?
1
Not at all
2
3
4
5
Perfectly
17
Q

Assessment of febrile children?

A
T: electronic thermometer in the axilla if <4w or
with infra-red tympanic thermometer
HR
RR
CRT
Signs of dehydration: skin turgor
How well did you know this?
1
Not at all
2
3
4
5
Perfectly
18
Q

Mx of child at “green” on risk stratificiation for

feverish illness?

A

Managed at home with appropriate care

advice, including when to seek further help

How well did you know this?
1
Not at all
2
3
4
5
Perfectly
19
Q

Mx of child at “amber” on risk stratificiation for

feverish illness?

A

Safety net or refer to paediatric specialist for
further assessment
Safety net: verbal/written info about warning
symptoms and how to access further care

How well did you know this?
1
Not at all
2
3
4
5
Perfectly
20
Q

Mx of child at “red” on risk stratificiation for

feverish illness?

A

Admit to hospital

How well did you know this?
1
Not at all
2
3
4
5
Perfectly
21
Q

A 3-year-old girl is brought in by her mother.
Her mother reports that she has been eating
less and refusing food for the past few weeks.
Despite this her mother has noticed that her
abdomen is distended and she has developed a
‘beer belly’. For the past year she has opened
her bowels around once every other day,
passing a stool of ‘normal’ consistency. There
are no urinary symptoms. On examination she
is on the 50th centile for height and weight.
Her abdomen is soft but slightly distended and
a non-tender ballotable mass can be felt on the
left side. Her mother has tried lactulose but
there has no significant improvement. What is
the most appropriate next step in
management?
Switch to polyethylene glycol 3350 +
electrolytes (Movicol Paediatric Plain) and
review in two weeks
Speak to a local paediatrician
Reassure normal findings and advise Health
Visitor review to improve oral intake
Prescribe a Microlax enema
Continue lactulose and add ispaghula husk
sachets

A

The history of constipation is not particularly
convincing. A child passing a stool of normal
consistency every other day is within the
boundaries of normal. The key point to this
question is recognising the abnormal
examination finding - a ballotable mass
associated with abdominal distension. Whilst
an adult with such a ‘red flag’ symptom/sign
would be fast-tracked it is more appropriate to
speak to a paediatrician to determine the best
referral pathway, which would probably be
clinic review the same week.

How well did you know this?
1
Not at all
2
3
4
5
Perfectly
22
Q

Wilms’ tumour

A

Wilms’ nephroblastoma is one of the most
common childhood malignancies. It typically
presents in children under 5 years of age, with
a median age of 3 years old

How well did you know this?
1
Not at all
2
3
4
5
Perfectly
23
Q

Features of Wilm’s tumour

A
Abdominal mass (most common PC)
Painless haematuria
Flank pain
Anorexia, fever
Unilateral in 95%
Mest found in 20%
How well did you know this?
1
Not at all
2
3
4
5
Perfectly
24
Q

Mx of Wilm’s

A
Management
nephrectomy
chemotherapy
radiotherapy if advanced disease
prognosis: good, 80% cure rate
How well did you know this?
1
Not at all
2
3
4
5
Perfectly
25
Q

Features of pertussis

A

Caused by Bordetella pertussis
10-14d incubation

Infants rountely immunised at 2,3,4m and 3-5y.
Pregnant women also immunised

How well did you know this?
1
Not at all
2
3
4
5
Perfectly
26
Q

Clinical features of pertussis

A

Coughing bouts: usually worse at night and
after feeding, may be ended by vomiting and
associated central cyanosis
inspiratory whoop (not always present), caused
by forced inspriration against a closed glottis
Persistent coughing may cause subconjunctival
haemorrhages or anorexia, leading to syncope
and seizures
Symptoms may last 10-14w and tend to be
more severe in infants
Lymphocytosis

How well did you know this?
1
Not at all
2
3
4
5
Perfectly
27
Q

Dx of pertussis

A

Per nasal swab culture for B. pertussis

PCR and serology may also be used

How well did you know this?
1
Not at all
2
3
4
5
Perfectly
28
Q

Mx of pertussis

A

Oral erythromycin to eradicate organism and
reduce spread
Has not been shown to alter the course of the
illness

How well did you know this?
1
Not at all
2
3
4
5
Perfectly
29
Q

A mother presents to your GP surgery with her
six month old daughter. She has been struggling

to feed her daughter, and her health visitor
found that she was small for her age. Her
mother is exhausted as she says her daughter
sleeps poorly.
On examination, the baby is just below the 3rd
centile in length. She has epicanthic folds and
low set ears. Her neck appears short and she
has micrognathia. You hear an ejection systolic
murmur on auscultation.
What is the most likely diagnosis?
Fragile X syndrome
Down’s syndrome
Patau syndrome
Klinefelter’s syndrome
Turner’s syndrome

A

Turner’s syndrome is a genetic condition due to
a loss or abnormality of one X chromosome. In
infancy, children often have difficulty with
feeding which contributes to poor weight gain,
although the often have short stature too when
older. Babies with Turner’s syndrome often
have multiple dysmorphic features, but a
webbed neck is often classical. It is also
associated with cardiac abnormalities, in this
question aortic stenosis although others are
also common. Chromosome analysis would be
needed to confirm the diagnosis.
While Down’s syndrome babies would have
many of the dysmorphic features, they would
not usually have a webbed neck or
micrognathia. They may have loose skin at the
nape of the neck but not webbing. It is caused
by Trisomy 21.
Klinefelter’s syndrome is caused by having an
extra X chromosome. They are often tall in
stature with small testes and gynaecomastia.
They do not tend to have the dysmorphic
features.

Fragile X syndrome is due to a CGG repeat on
the X chromosome. They tend to have learning
difficulties, long ears, mitral valve prolapse and
a large forehead and jaw.
Patau’s syndrome is caused by trisomy 13. They
do tend to have intrauterine growth restriction
leading to low birth weight, and can have
congenital heart defects and ear abnormalities.
However, they do not have webbing of the
neck, and eye dysmorphic features tend to be
microphthalmia or anophthalmia. They
typically have rocker bottom feet and
polydactyly

How well did you know this?
1
Not at all
2
3
4
5
Perfectly
30
Q

9

Features of Turner’s syndrome

A
45XO
Short stature
Shield chest, widely spaced nipples
Webbed neck
Cardiac: bicuspid aortic valve, coarctation
Primary amenorrhoea
Cystic hygroma
High-arched palate
Short fourth metacarpal
Multiple pigmented naevi
Lymphoedema
Increased incidence of autoimmune disease:
thyroiditis and Crohn's especially
How well did you know this?
1
Not at all
2
3
4
5
Perfectly
31
Q

A 2 year old boy presents to the GP with his
mother. She is worried that he is not growing at
the same rate as the other children at his play
group. His mother describes foul smelling
diarrhoea about 4-5 times a week,
accompanied by abdominal pain.
On examination he has a bloated abdomen and
wasted buttocks. He has dropped 2 centile lines
and now falls on the 10th centile.

What is the most appropriate initial
investigation?
Stool sample
IgA TTG antibodies
Hydrogen breath test
Endoscopy
Abdominal xray
A

The most likely diagnosis here is coeliac
disease, diagnosed using IgA TTG antibodies, as
explained below.
A stool sample would be diagnostic for
gastroenteritis, in order to dictate which
antibiotic should be used.
The hydrogen breath test is used to diagnose
irritable bowel syndrome or some food
intolerances.
Endoscopy is more commonly used in adults
where cancer is suspected.
An abdominal X-ray may be useful where
obstruction is suspected.
Coeliac disease is a digestive condition which is
becoming increasingly common, and describes
an adverse reaction to gluten. gluten is a
protein found in wheat, barley and rye.

How well did you know this?
1
Not at all
2
3
4
5
Perfectly
32
Q

Features of coeliac in children?

A
May coincide with the introduction of cereals
FTT
Diarrhoea
Abdominal distension
Older children may present with anaemia
May not be dxed until adulthood
How well did you know this?
1
Not at all
2
3
4
5
Perfectly
33
Q

Hirschprung’s features

A

Hirschsprung’s disease is caused by an
aganglionic segment of bowel due to a
developmental failure of the parasympathetic
Auerbach and Meissner plexuses. Although rare
(occurring in 1 in 5,000 births) it is an important
differential diagnosis in childhood constipation

Possible presentations
neonatal period e.g. failure or delay to pass
meconium
older children: constipation, abdominal
distension
Associations
3 times more common in males
Down's syndrome
How well did you know this?
1
Not at all
2
3
4
5
Perfectly
34
Q

Erb’s palsy

A

Erb’s palsy occurs due to damage to the upper
brachial plexus most commonly from shoulder
dystocia. Damage to these nerve roots results
in a characteristic pattern: adduction and
internal rotation of the arm, with pronation of
the forearm. This classic physical position is
commonly called the ‘waiter’s tip’.

How well did you know this?
1
Not at all
2
3
4
5
Perfectly
35
Q

Klumpke’s palsy

A

Klumpke’s palsy occurs due to damage of the
lower brachial plexus and commonly affects the
nerves innervating the muscles of the hand.

How well did you know this?
1
Not at all
2
3
4
5
Perfectly
36
Q

A male child from a travelling community is
diagnosed with measles. Which one of the
following complications is he at risk from in the
immediate aftermath of the initial infection?
Arthritis
Pancreatitis
Infertility
Subacute sclerosing panencephalitis
Pneumonia

A

Subacute sclerosing panencephalitis is seen but
develops 5-10 years following the illness.
Pancreatitis and infertility may follow mumps
infection

Pneumonia

How well did you know this?
1
Not at all
2
3
4
5
Perfectly
37
Q

Measles clinical features

A

Prodrome: irritable, conjunctivitis, fever
Koplik spots: grains of salt, on buccal mucosa
Rash: starts behind ears, then to whole body,
discrete maculopapular rash becoming blotchy
and confluent

How well did you know this?
1
Not at all
2
3
4
5
Perfectly
38
Q

Cxs of measles

A

encephalitis: typically occurs 1-2 weeks
following the onset of the illness)
subacute sclerosing panencephalitis: very rare,
may present 5-10 years following the illness
febrile convulsions
giant cell pneumonia

keratoconjunctivitis, corneal ulceration
diarrhoea
increased incidence of appendicitis
myocarditis

How well did you know this?
1
Not at all
2
3
4
5
Perfectly
39
Q

Mx of measles contacts

A

if a child not immunized against measles comes
into contact with measles then MMR should be
offered (vaccine-induced measles antibody
develops more rapidly than that following
natural infection)
this should be given within 72 hours

How well did you know this?
1
Not at all
2
3
4
5
Perfectly
40
Q

Def: nephrotic syndrome

A
Nephrotic syndrome is classically defined as a
triad of
proteinuria (> 1 g/m^2 per 24 hours)
hypoalbuminaemia (< 25 g/l)
oedema
How well did you know this?
1
Not at all
2
3
4
5
Perfectly
41
Q

Nephrotic syndrome in children

A

In children the peak incidence is between 2 and
5 years of age. Around 80% of cases in children
are due to a condition called minimal change
glomerulonephritis. The condition generally
carries a good prognosis with around 90% of
cases responding to high-dose oral steroids.
Other features include hyperlipidaemia, a
hypercoagulable state (due to loss of
antithrombin III) and a predisposition to
infection (due to loss of immunoglobulins)

How well did you know this?
1
Not at all
2
3
4
5
Perfectly
42
Q

Features of acute epiglottitis

A

Acute epiglottitis is rare but serious infection
caused by Haemophilus influenzae type B.
Prompt recognition and treatment is essential
as airway obstruction may develop. Epiglottitis

was generally considered a disease of
childhood but in the UK it is now more
common in adults due to the immunisation
programme. The incidence of epiglottitis has
decreased since the introduction of the Hib
vaccine

How well did you know this?
1
Not at all
2
3
4
5
Perfectly
43
Q

Clinical features of epiglottit

A
Features
rapid onset
high temperature, generally unwell
stridor
drooling of saliva
How well did you know this?
1
Not at all
2
3
4
5
Perfectly
44
Q

A 7-year-old boy is brought in to the GP surgery
with an exacerbation of asthma. On
examination he has a bilateral expiratory
wheeze but there are no signs of respiratory
distress. His respiratory rate is 36 / min and PEF
around 60% of normal. What is the most
appropriate action with regards to steroid
therapy?
Oral prednisolone for 3 days
Admit for intravenous steroids
Give a stat dose of oral dexamethasone
Double his usual beclometasone dose
Do not give steroids

A

Oral prednisolone for 3 days

How well did you know this?
1
Not at all
2
3
4
5
Perfectly
45
Q

Mx of mild-moderate acute asthma

A

Bronchodilator:
Beta-2 agonist via a spacer (<3y use a closefitting
mask)
1 puff every 15-30secs, up to a maximum of 10
puffs, repeat dose after 10-20 mins if
necessary
If symptoms are not controlled, repeat beta-2
and refer to hospital
Steroid therapy:
should be given to all children with asthma
exacerbation
Treatment for 3-5d

How well did you know this?
1
Not at all
2
3
4
5
Perfectly
46
Q
A 9-year-old boy is brought to surgery with
recurrent headaches. What is the most
common cause of headaches in children?
Migraine
Depression
Refractive errors
Tension-type headache
Cluster headache
A

Migraine

How well did you know this?
1
Not at all
2
3
4
5
Perfectly
47
Q

Features of Hand foot and mouth disease?:

A

Hand, foot and mouth disease is a self-limiting
condition affecting children. It is caused by the
intestinal viruses of the Picornaviridae family
(most commonly coxsackie A16 and enterovirus
71). It is very contagious and typically occurs in
outbreaks at nursery
Clinical features
mild systemic upset: sore throat, fever
oral ulcers
followed later by vesicles on the palms and
soles of the feet

How well did you know this?
1
Not at all
2
3
4
5
Perfectly
48
Q

Mx of hand foot and mouth?

A

Management
general advice about hydration and analgesia
reassurance no link to disease in cattle
children do not need to be excluded from
school*
*The HPA recommends that children who are
unwell should be kept off school until they feel
better. They also advise that you contact them
if you suspect that there may be a large
outbreak.

How well did you know this?
1
Not at all
2
3
4
5
Perfectly
49
Q

Features of Croup?

A

Croup is a form of upper respiratory tract
infection seen in infants and toddlers. It is
characterised by stridor which is caused by a
combination of laryngeal oedema and
secretions. Parainfluenza viruses account for
the majority of cases.
Epidemiology
peak incidence at 6 months - 3 years
more common in autumn
Features
stridor
barking cough (worse at night)
fever
coryzal symptoms

How well did you know this?
1
Not at all
2
3
4
5
Perfectly
50
Q

Mx of croup

A
Single dose of oral dexamethasone
(0.15mg/kg)
Emergency:
High flow O2
Nebulised adrenaline
How well did you know this?
1
Not at all
2
3
4
5
Perfectly
51
Q

The parents of a 14-month-old girl present to
their GP. They have noticed that in some
photos there is no ‘red eye’ on the left hand
side. When you examine the girl you notice an
esotropic strabismus and a loss of the redreflex
in the left eye. There is a family history of
a grandparent having an enucleation as a child.
What is the most likely diagnosis?
Congenital hypertrophy of the retinal pigment
epithelium

Uveal malignant melanoma
Neuroblastoma
Retinoblastoma
Congenital cataract

A

A congenital cataract may cause a loss of the
red-reflex but is likely to have been detected at
birth or during the routine baby-checks. It
would also not explain the family history of
enucleation.
Retinoblastoma

How well did you know this?
1
Not at all
2
3
4
5
Perfectly
52
Q

Features of retinoblastoma

A
Retinoblastoma is the most common ocular
malignancy found in children. The average age
of diagnosis is 18 months.
Pathophysiology
caused by a loss of function of the
retinoblastoma tumour suppressor gene on
chromosome 13
around 10% of cases are hereditary
Possible features
absence of red-reflex, repalced by a white pupil
(leukocoria) - the most common presenting
symptom
strabismus
visual problems
How well did you know this?
1
Not at all
2
3
4
5
Perfectly
53
Q

Mx of retinoblastoma

A

Management
enucleation is not the only option
depending on how advanced the tumour is
other options include external beam radiation
therapy, chemotherapy and photocoagulation
Prognosis
excellent, with > 90% surviving into adulthood

How well did you know this?
1
Not at all
2
3
4
5
Perfectly
54
Q

What differentiates between NRDS and TTN?

A

Neonates with NRDS usually present with
respiratory distress shortly after birth which
usually worsens over the next few days. In
contrast, TTN usually presents with tachypnoea
shortly after birth and often fully resolves
within the first day of life. A chest radiograph
can be useful

How well did you know this?
1
Not at all
2
3
4
5
Perfectly
55
Q

CXR in NRDS?

A

Diffuse ground glass lungs
Low volumes
Bell shaped throax

How well did you know this?
1
Not at all
2
3
4
5
Perfectly
56
Q

CXR in TTN

A
Heart failure type pattern
Intersitital oedema
PLeural effusions
But normal heart size in contrast to congenital
heart disease
How well did you know this?
1
Not at all
2
3
4
5
Perfectly
57
Q

Features of Surfactant lung disease?

A

Surfactant deficient lung disease (SDLD, also
known as respiratory distress syndrome and
previously as hyaline membrane disease) is a
condition seen in premature infants. It is
caused by insufficient surfactant production
and structural immaturity of the lungs
The risk of SDLD decreases with gestation
50% of infants born at 26-28 weeks
25% of infants born at 30-31 weeks
Other risk factors for SDLD include
male sex
diabetic mothers
Caesarean section
second born of premature twins

Clinical features are those common to
respiratory distress in the newborn, i.e.
tachypnoea, intercostal recession, expiratory
grunting and cyanosis
Chest x-ray characteristically shows ‘groundglass’
appearance with an indistinct heart
border

How well did you know this?
1
Not at all
2
3
4
5
Perfectly
58
Q

Mx of SDLD?

A

Management
prevention during pregnancy: maternal
corticosteroids to induce fetal lung maturation
oxygen
assisted ventilation
exogenous surfactant given via endotracheal
tube

How well did you know this?
1
Not at all
2
3
4
5
Perfectly
59
Q

What timing indicates idiopathic constipation?

A

Starts after a few weeks of life
Obvious precipitating factors coinciding with
the start of symptoms: fissure, change of diet,
timing of potty/toilet training or acute events
such as infections, moving house, starting
nursery/school, fears and phobias, major
change in family, taking medicines

How well did you know this?
1
Not at all
2
3
4
5
Perfectly
60
Q

Mx of feacal impaction

A

Polyethylene glycoe 3350 + electroyles (using
an escalating dose) is first line
Stimulant laxative can be addied if first line
does not lead to disimpaction after 2 weeks.
Subsititue a stimulant laxative singly or in
combination with an osmotic laxative such as

lactulose if Movicol Paediatric plan is not
tolerated.
Inform families that disimpaction treatment
can initially increase symptoms of soiling and
abdominal pain

How well did you know this?
1
Not at all
2
3
4
5
Perfectly
61
Q

General points in Mx of constipation?

A

do not use dietary interventions alone as firstline
treatment although ensure child is having
adequate fluid and fibre intake
consider regular toileting and non-punitive
behavioural interventions
for all children consider asking the Health
Visitor or Paediatric Continence Advisor to help
support the parents.

How well did you know this?
1
Not at all
2
3
4
5
Perfectly
62
Q

Mx of infants not yet weaned with

constipation

A

bottle-fed infants: give extra water in between
feeds. Try gentle abdominal massage and
bicycling the infant’s legs
breast-fed infants: constipation is unusual and
organic causes should be considered

How well did you know this?
1
Not at all
2
3
4
5
Perfectly
63
Q

Features of:

Chickenpox

A

Fever initially
Rash starting on head/trunk before spreading
Initially macular, then papular, then vessciular
Normally mild systemic upset

How well did you know this?
1
Not at all
2
3
4
5
Perfectly
64
Q

Features of mumps

A

Fever, malaise, muscular pain
Parotitis initially unilateral becoming bilateral in
70%

How well did you know this?
1
Not at all
2
3
4
5
Perfectly
65
Q

Features of measles

A

Prodrome: irritable, conjuncitivits, fever
Koplik spots

Rash starting behind ears, spreading to the
whole body
Initially discrete maculopapular rash that
becomes blotchy and confluent

How well did you know this?
1
Not at all
2
3
4
5
Perfectly
66
Q

Features of Rubella

A

Pink maculopapular rash initially on face before
spreading to the whole body, usually faind by
the 3-5th day
Suboccipital and postauricular
lymphadenopathy

How well did you know this?
1
Not at all
2
3
4
5
Perfectly
67
Q

Features of erythema infectiosum

A
AKA slapped cheek syndrome
Caused by parvovirus B19
Lethargy, fever, headache
Slapped cheek rash spreading to proximal arms
and extensor surfaces
How well did you know this?
1
Not at all
2
3
4
5
Perfectly
68
Q

Features of Scarlet fever

A
Reaction to erythrogenci toxins produced by
group A haemolytic strep
Fever, malaise, tonsilitis
Strawberry tongue
Fine punctate erythema sparing face
How well did you know this?
1
Not at all
2
3
4
5
Perfectly
69
Q

Features of hand, foot and mouth disease

A

Caused by coxsackie A16 virus
Mild systemic upset: sore throat, fever
Vesciles in the mouth and on the palms and
soles of the feet

How well did you know this?
1
Not at all
2
3
4
5
Perfectly
70
Q

Scarlet fever features

A

Scarlet fever is a reaction to erythrogenic toxins
produced by Group A haemolytic streptococci
(usually Streptococcus pyogenes). It is more
common in children aged 2 - 6 years with the
peak incidence being at 4 years.
Scarlet fever has an incubation period of 2-4
days and typically presents with:
fever
malaise
tonsillitis
‘strawberry’ tongue
rash - fine punctate erythema (‘pinhead’) which
generally appears first on the torso and spares

the face although children often have a flushed
appearance with perioral pallor. The rash often
has a rough ‘sandpaper’ texture.
Desquamination occurs later in the course of
the illness, particularly around the fingers and
toes

How well did you know this?
1
Not at all
2
3
4
5
Perfectly
71
Q

Dx of scarlet fever

A

Throat swab usually taken but antibiotic

treatment should be commenced immediately

How well did you know this?
1
Not at all
2
3
4
5
Perfectly
72
Q

Mx of scarlet fever

A

Oral penicillin V (penallergic: azithromycin)
Children can return to school 24h after
commencing antibiotics
Notifiable disease

How well did you know this?
1
Not at all
2
3
4
5
Perfectly
73
Q

Cx of Scarlet fever?

A

Otitis media: most common
Rheumatic fever: typically 20d after infection
Acute GN

How well did you know this?
1
Not at all
2
3
4
5
Perfectly
74
Q

Characteristic symptoms in ADHD?

A

Extreme restlessness
Poor concentration
Uncontrolled activity
Impusliveness

How well did you know this?
1
Not at all
2
3
4
5
Perfectly
75
Q

Patau syndrome (trisomy 13)

A

Microcephalic, small eyes
Cleft lip/palate
Polydactyly
Scalp lesions

How well did you know this?
1
Not at all
2
3
4
5
Perfectly
76
Q

Edward’s syndrome (trisomy 18)

A

Micrognathia
Low-set ears
Rocker bottom feet
Overlapping of fingers

How well did you know this?
1
Not at all
2
3
4
5
Perfectly
77
Q

Fragile X

A
Learning difficulties
Macrocephaly
Long face
Large ears
Macro-orchidism
How well did you know this?
1
Not at all
2
3
4
5
Perfectly
78
Q

Noonan syndrome

A

Webbed neck
Pectus excavatum
Short stature
Pulmonary stenosis

How well did you know this?
1
Not at all
2
3
4
5
Perfectly
79
Q

Prader-Willi syndrome

A

Hypotonia
Hypogonadism
Obesity

How well did you know this?
1
Not at all
2
3
4
5
Perfectly
80
Q

William’s syndrome

A
Short stature
Learning difficulties
Friendly, extrovert personality
Transient neonatal hypercalcaemia
Supravalvular aortic stenosis
How well did you know this?
1
Not at all
2
3
4
5
Perfectly
81
Q

What Ixs should be performed in infants <3m

old with fever?

A
FBC
Blood culture
CRP
Urine dip
CXR if respiratory signs are present
Stool culture if diarrhoea is present
How well did you know this?
1
Not at all
2
3
4
5
Perfectly
82
Q

What are hte criteria for admission in

bronchiolitis?

A

Apnoea (observed or reported)
Persistent oxygen saturation of <92% in air
Inadequate oral fluid intake (<50% of normal
fluid intake)
Persisting severe respiratory distress, for
example grunting, marked chest recession, or a
respiratory rate of over 70 breaths/minute.

How well did you know this?
1
Not at all
2
3
4
5
Perfectly
83
Q

Mx of RSV

A

Admit if fulfils criteria
Deliver humidifed oxygen best through head
box. Level can be determiend with pulse
oximetry.
Fluids and feed may need to be given by NG
tube or IV.
Only 5% require venitlation.

How well did you know this?
1
Not at all
2
3
4
5
Perfectly
84
Q

A 2-year-old girl is brought to her GP because
her mother has noticed she is constantly itching
her bottom at night. Her mother says she has
noticed some strange looking white bits when
she wipes her daughters bottom following a
bowel motion. What is the most appropriate
management option?
Prescribe 14 days of daily miconazole for whole
household and issue hygiene advice.
Issue hygiene advice only.
Prescribe a single dose of mebendazole for the
daughter and issue hygiene advice.
Prescribe a single dose of mebendazole for the
whole household and issue hygiene advice

A

This child is highly likely to have a threadworm
infection with symptoms of perianal itching
that is worse at night. It is also possible to see
threadworms, described as small threads of
slowly-moving white cotton either around the
anus or in the stools.
The risk of transmission in families is as high as
75%, and asymptomatic infestation is common.
For this reason an anthelmintic drug
(mebendazole) should be given as a single dose
to all household members.

How well did you know this?
1
Not at all
2
3
4
5
Perfectly
85
Q

64

Features of threadworm infection

A

Asymptomactic in 90%
Perianal itching, particulrly at night.
Girls may have vulval symptoms.
Dx can be made by applyoing sellotape to the
perianal area and sending it to the laboratory
for microscopy to identify the eggs.

How well did you know this?
1
Not at all
2
3
4
5
Perfectly
86
Q

A mother comes to surgery with her 6-year-old
son. During the MMR scare she decided not to
have her son immunised. However, due to a
recent measles outbreak she asks if he can still
receive the MMR vaccine. What is the most
appropriate action?
Arrange for measles immunoglobulin to be
given
Cannot vaccinate at this age as live vaccine
Give separate measles vaccine
Give MMR with repeat dose in 3 months
Give MMR with repeat dose in 5 years

A

The Green Book recommends allowing 3
months between doses to maximise the
response rate. A period of 1 month is
considered adequate if the child is greater than
10 years of age. In an urgent situation (e.g. an
outbreak at the child’s school) then a shorter
period of 1 month can be used in younger
children.

How well did you know this?
1
Not at all
2
3
4
5
Perfectly
87
Q

Transient synovitis

A

Acute onset
Usually accompanies viral infections, but the
child is well or has a mild fever
More common in boys, aged 2-12 years

How well did you know this?
1
Not at all
2
3
4
5
Perfectly
88
Q

Septic arthritis/osteomyelitis

A

Unwell child, high fever

How well did you know this?
1
Not at all
2
3
4
5
Perfectly
89
Q

Development dysplasia of the hip

A

Usually detected in neonates

6 times more common in girls

90
Q

Perthes disease

A

More common at 4-8 years

Due to avascular necrosis of the femoral head

91
Q

myotonic dystrophy

A

Myotonic dystrophy (dystrophia
myotonica, myotonia atrophica) is a chronic,
slowly progressing, highly variable, inherited
multisystemicdisease. It is an autosomaldominant
disease.It is characterized by wasting
of the muscles (muscular dystrophy), cataracts,
heart conduction defects, endocrine changes,
and myotonia.[1]

There are two main types of myotonic
dystrophy. Myotonic dystrophy type 1 (DM1),
also called Steinert disease, has a severe
congenital form and an adult-onset form.
Myotonic dystrophy type 2 (DM2), also called
proximal myotonic myopathy (PROMM) is rarer
than DM1 and generally manifests with milder
signs and symptoms. Myotonic dystrophy can
occur in people of any age. Both forms of the
disease display an autosomal-dominant pattern
of inheritance. Both “DM1” and “DM2” have
adult-onset forms.

92
Q

Which one of the following statements
regarding scabies is false?
All members of the household should be
treated
Typically affects the fingers, interdigital webs
and flexor aspects of the wrist in adults
Scabies causes a delayed type IV
hypersensitivity reaction
Patients who complain of pruritus 4 weeks
following treatment should be retreated
Malathion is suitable for the eradication of
scabies

A

It is normal for pruritus to persist for up to 4-6

weeks post eradication

93
Q

Features opf Scabies

A

Scabies is caused by the mite Sarcoptes scabiei
and is spread by prolonged skin contact. It
typically affects children and young adults.
The scabies mite burrows into the skin, laying
its eggs in the stratum corneum. The intense
pruritus associated with scabies is due to a
delayed type IV hypersensitivity reaction to
mites/eggs which occurs about 30 days after
the initial infection.

94
Q

Clinical features of scabies

A

Widespread prutirus
Linear burrows on the side of fingers,
interdigital webs and flexor aspects of hte
wrist.
Infants the face and scalp may be affected.
2o features are due to scratching excoriation,
infection

95
Q

Mx of scabies

A

Permethrin 5% is first line
Malathion is second line.
Guidance on use.
Pruritus persists 4-6w post eradication

96
Q

When is crusted scabies seen?

Mx

A

Crusted scabies is seen in patients with
suppressed immunity, especially HIV.
The crusted skin will be teeming with hundreds
of thousands of organisms.
Ivermectin is the treatment of choice and
isolation is essentia

97
Q

A 3-year-old girl presents with a 3 day history of
fever and bloody diarrhoea. Over the past 24
hours she has had 5 episodes of loose bloody
stools. On examination she has a temperature
of 39.6ºC, a heart rate of 175 bpm and her
abdomen is soft with generalised tenderness. It
is also noted that she has a reduced urinary
output. Blood tests show a haemolytic anaemia
and raised urea.
What is the most likely diagnosis?
Campylobacter gastroenteritis
Salmonella gastroenteritis
Norovirus
Rotavirus
Escherichia coli gastroenteritis

A

A short history of bloody diarrhoea is very
suggestive of haemorrhagic gastroenteritis
which can occur due to a variety of pathogens
including Campylobacter, Salmonella and
Escherichia coli.
In this case, the haemolytic anaemia and raised
urea suggest haemolytic uraemic syndrome.
Haemolytic anaemia and renal failure form two
parts of the classic triad of haemolytic uraemic
syndrome. The third part of the triad is
thrombocytopenia. It is usually caused by
Escherichia coli subtype 0157. Treatment is
supportive as antibiotics are contraindicated.

98
Q

A 9 year old boy is brought to the emergency
department by ambulance. For approximately
24 hours he has had nausea and vomiting.
However, he has now developed acute
abdominal pain and when he arrives in the
emergency department his breathing is
laboured, deep and of a gasping nature. He is
usually fit and well and is not prescribed any
medication. Blood results show the following:
Na+130 mmol/l
K+3.5 mmol/l
HCO3-19 mmol/l
What is the likely cause?
Sepsis
Rotavirus
Intestinal obstruction
Meningitis
Diabetic ketoacidosis

A

The patient in this scenario has developed
diabetic ketoacidosis (DKA). The important
pieces of information to consider when
answering this question are his acute
presentation and the blood results.
This patient has presented to the emergency
department with nausea, vomiting and acute

abdominal pain. These are all symptoms of
diabetic ketoacidosis. Furthermore, the
laboured, deep breathing that is mentioned is
Kussmaul’s breathing, which is witnessed in
DKA and metabolic acidosis. Kussmaul’s
breathing occurs whereby excess CO2 is
exhaled as a compensatory mechanism for an
increased blood pH. The recognition of
Kussmauls breathing in this question is one of
the major factors in getting this question
correct, as you would not expect to see this
phenomenon in the other 4 possible answers.
The blood results are concurrent with a
diagnosis of DKA. Bloods will often show a
hyponatraemia, low bicarbonate and a
hypokalaemia in severe cases. The low
bicarbonate in this question gives the
indication that there is an acidosis in this
patient, which helps in deriving the correct
answer.
Taking into account all other answers, they
each could explain some of the symptoms of
this child. However, the low bicarbonate, his
symptoms and Kussmaul’s respirations should
lead to a working diagnosis of diabetic
ketoacidosis in this patient.

99
Q

Features of DKA

A

Abdo pain
Polyuria, polydipsia, dehydration
Kussmaul respiration
Acetone smelling breath

100
Q

A baby boy born 6 hours ago has an APGAR
score of 10. He is not cyanosed, has a pulse of
140, cries on stimulation, his arms and legs
resist extension and he has a good cry, He
appears jaundiced. What is the most
appropriate action?
Encourage the mother to sit with the baby in
sunlight
Arrange a blood transfusion
Start phototherapy

Prescribe intravenous immunoglobulin
Measure and record the serum bilirubin level
urgently

A

Measure and record the serum bilirubin level
urgently (within 2 hours) in all babies with
suspected or obvious jaundice in the first 24
hours of life since this is likely to be
pathological rather than physiological jaundice.
NICE CG98

101
Q

Causes of jaundice in the first 24h

A

Rhesus disease
ABO disease
Hereditary spherocytosis
G6PDD

102
Q

Jaundice <24h y/o

A

Always pathological

103
Q

Jaundice 2-14d

A

Common and usually physiological, commonly

seen in breast fed babies

104
Q

Causes of prolonged jaundice (>14d)

A
Biliary atresia
Hypothyroidism
Galactosaemia
HTI
Breast milk jaundice
Congenital infections e.g. CMV, toxoplasmosis
105
Q

Daniel is a newborn who is having his baby
check done by nurse Karen, who notices that
he has microcephaly with a prominent occiput,
low set ears, micrognathia, palpebral fissures

and wide spaced eyes. What genetic disorder
are these features suggestive of?
Edward's syndrome
Down's syndrome
Turner's syndrome
Noonan syndrome
Angelman syndrome
A
The correct answer for this question is Edward's
syndrome.
All of the aforementioned characteristics can
be present in Edward's syndrome.
Furthermore, individual's with Edward's
syndrome can also have:
Ptosis
Rocker bottom feet
Undescended testes
106
Q

Features of kawasaki’s disease

A
High-grade fever which lasts >5d and is
characteristically resistant to antipyretics
Conjunctival injection
Bright red, cracked lips
Strawberry tongue
Cervical lymphadenopathy
Red plams of the hands and the soles of the
feet which alter peel
107
Q

Mx of kawasaki

A

High dose aspirin
IVIG
Echocardiogram as screening test for coronary
artery aneurysm

108
Q

Cx of kawasaki

A

Coronary artery aneurysm

109
Q

An 8-year-old boy presents with increasing
jaundice over the past week. He was recently
treated with nitrofurantoin for a simple urinary
tract infection. On examination he is obviously
jaundiced, and he is looking pale and
breathless. Investigation results are as follows:
Hb58 g/l
Platelets250 * 109/l
WBC6.5 * 109/l
A blood films demonstrates red cell fragments
and Heinz bodies.
What is the most likely diagnosis?
Pyruvate kinase deficiency
Sickle cell disease
Glucose-6-phosphate dehydrogenase
deficiency
Beta-thalassaemia
Hereditary spherocytosis

A

Glucose-6-phosphate dehydrogenase
deficiency is an X linked disorder affecting red
cell enzymes. It results in a reduced ability of
the red cells to respond to oxidative stress.
Therefore, red cells have a shorter life span and
are more susceptible to haemolysis, particularly
in response to drugs (e.g. nitrofurantoin),
infection, acidosis and certain dietary agents
(e.g. fava beans).

The red cell fragments, Heinz bodies and
anaemia confirm a haemolytic anaemia.

110
Q

Features of G6PD

A
Features
neonatal jaundice is often seen
intravascular haemolysis
gallstones are common
splenomegaly may be present
Heinz bodies on blood films
111
Q

A 6-year-old boy is reviewed in clinic due to
nocturnal enuresis. His mother has tried using a
star-chart but unfortunately this has not
resulted in any significant improvement. Of the
following options, what is the most appropriate
initial management strategy?
Enuresis alarm
Trial of oral desmopressin
Trial of imipramine
Trial of intranasal desmopressin
Restrict fluids in the afternoon and evening

A

Restricting fluids is not recommended advice -
Clinical Knowledge Summaries suggest: ‘Do not
restrict fluids. The child should have about
eight drinks a day, spaced out throughout the
day, the last one about 1 hour before bed.’
Enuresis alarm

112
Q

Mx of Nocturnal enuresis

A

Look for possible underlying causes/triggers:
constipation, DM, UTI
Advise on fluid intake
Reward systems e.g. star charts
Enuresis alarm for children <7
Desmopressin may be used if >7y particulalry if
ST control is needed or an enuresis alarm has
been ineffective

113
Q

Henry is a 29 week premature baby who was
born 2 weeks ago. Over the past week it has
been noted that he has had bloody stool,
abdominal distension and has not been feeding
well. Physical examination reveals an increased
abdominal girth with reduced bowel sounds.
Abdominal X-ray shows dilated asymmetrical
bowel loops and bowel wall oedema. What is
the likely diagnosis?
Intussusception
Inflammatory bowel disease
Pyloric stenosis
Hirschsprung’s disease
Necrotising enterocolitis

A

The correct answer for this question is

necrotising enterocolitis.

114
Q

You see a worried mum with her 6 month old
baby boy. She is concerned that his skull shape
is not normal. His development and birth have
been normal and there are no conditions in the
family. On examination his head circumference
is at the 40th centile with his height and weight
at the 30th centile. His occiput is flattened on
the left, his left ear mildly protruding forward
and his left forehead more prominent than the
right. No other abnormality is detected. What
is the most appropriate management?
Urgent referral to neurosurgery
Suggest buying an infant helmet
Arrange an MRI scan
Routine referral to community child health
clinic
Reassurance

A

Plagiocephaly is more common since there
have been campaigns to encourage babies to
sleep on their back to reduce the risk of sudden
infant death syndrome (SIDS). Plagiocephaly is
a skull deformity producing unilateral occipital
flattening, which pushes the ipsilateral
forehead ear forwards producing a
‘parrallelogram’ appearance. The vast majority
improve by age 3-5 due to the adoption of a
more upright posture. Helmets are not usually
recommended as there was no significant
difference between groups in a randomised
controlled trial. Turning the cot around may
help the child look the other way and take the
pressure off the one side. Other simple
methods include giving the baby time on their
tummy during the day, supervised supported
sitting during the day, and moving toys/

mobiles around in the cot to change the focus
of attention. Ensure all advice is in line with
prevention of SIDS

115
Q

A 4-year-old boy was discharged from the
hospital six weeks ago after an episode of viral
gastroenteritis. He now has 4-5 loose stools
each day which has been present for the past
four weeks.
What is the most likely diagnosis?
Coeliac disease
Inflammatory bowel disease
Secondary bacterial infection
Lactose intolerance
Clostridium difficile infection

A

Transient lactose intolerance is a common
complication of viral gastroenteritis. Removal of
lactose from the diet for a few months followed
by a gradual reintroduction usually resolves the
problem.

116
Q

A 1-year-old girl is investigated for recurrent
urinary tract infections. A micturating
cystourethrogram is ordered:
What does this image demonstrate?

Vesicoureteric reflux
Horseshoe kidney
Paediatric urolithiasis
Duplex collecting system
Isolated right-sided hydronephrosis
A

This image demonstrates grade V vesicoureteric
reflux - gross dilatation of the ureter, pelvis and
calyces with ureteral tortuosity. A DMSA scan is
needed to identify renal scarring.

117
Q

A 15-year-old boy presents to his GP
complaining of knee pain for one week. He has
no significant past medical history. Which of
the following would make a diagnosis of
Osgood-Schlatter disease more likely?
Bilateral knee pain.
Sudden onset of symptoms and acutely
painful.
Knee pain isolated to the posterior aspect of
the knee joint.
Pain relieved by rest and made worse by
kneeling and activity, such as running or
jumping.
Locking of the knee on movement

A

Osgood-Schlatter disease may be diagnosed on
the basis of clinical features alone. This age
group (adolescent) is the most likely age to
suffer from this condition and is localized to the
tibial tuberosity.
Typically, pain is:
Unilateral (but may be bilateral in up to 30% of
people).
Gradual in onset and initially mild and
intermittent, but may progress to become
severe and continuous.
Relieved by rest and made worse by kneeling
and activity, such as running or jumping.

118
Q

A 2-year-old boy is brought to the surgery by
his mother with earache and pyrexia. On
examination of the precordium a murmur is
heard. Which one of the following
characteristics is not consistent with an

innocent murmur?
Short buzzing murmur in the aortic area
Soft-blowing murmur in the pulmonary area
Varies with posture
Diastolic murmur
Continuous blowing noise heard just below the
clavicles

A

Diastolic murmur

119
Q
Which one of the following types of
glomerulonephritis is most characteristically
associated with streptococcal infection in
children?
Focal segmental glomerulosclerosis
Diffuse proliferative glomerulonephritis
Membranous glomerulonephritis
Mesangiocapillary glomerulonephritis
Rapidly progressive glomerulonephritis
A

Diffuse proliferative glomerulonephritis

120
Q

A mother brings her 3-year-old child in to
receive the DTP booster. Which one of the
following would make it inappropriate to give
the vaccination today?
Being below the 2nd centile for weight
Family history of allergy to DTP
Recent onset of a seizure disorder currently
being investigated
Planned general anaesthesia in 2 weeks time
Being born at 29 weeks gestation

A

DTP: vaccination should be deferred in children
with an evolving or unstable neurological
condition

121
Q

16-year-old boy presents to the emergency
room with a history of groin pain for the past
three hours. He has associated nausea and has
vomited three times. He reports that he
recently had unprotected vaginal sex. On
examination there is tenderness and swelling of
the scrotum and left testicle, with absence of
the cremaster reflex on the left side. Elevation
of the affected testicle causes increased pain.
What is the most likely diagnosis?
Torsion of the hydatid of Morgagni
Strangulated inguinal hernia
Epididymitis
Testicular torsion
Hydrocoele

A

Testicular torsion occurs when the testis turns
on the remnant of the processus vaginalis
thereby restricting blood flow. It usually
presents with acutely severe testicular pain
often with associated nausea and vomiting.
There may be swelling of the testis with
overlying erythema. The cremaster reflex may
also be absent on the affected side. Elevation of
the testicle often results in worsening of the
pain.
Although this patient recently had unprotected
sex, the history is less suggestive of
epididymitis. With epididymitis we would
expect urinary symptoms. In addition, elevation
of the testes often relieves the pain (Prehn’s
sign positive).

122
Q

You are called to the post natal ward to review
an 8 hour old baby born by elective caesarian
section at 39 weeks gestation. After reading the
case notes you discover the use of maternal
labetalol for high blood pressure. On
examination the baby appears jittery and
hypotonic. What is the most appropriate next
step?
Record temperature and ensure adequately
wrapped

Perform full septic screen
Measure blood glucose levels
Start empirical antibiotics for early onset sepsis
Re-examine after next feed

A

A jittery and hypotonic baby may suggest
neonatal hypoglycaemia. The use of maternal
labetalol is a risk factor and these babies must
have their blood glucose measured. Neonatal
abstinence syndrome may also present in this
way and so the use of maternal opiates or illicit
drug use in pregnancy should also be
ascertained.

123
Q

On routine antenatal swabs, a mother is found
to be colonised with Group B Streptococcus.
However, she did not receive adequate
intrapartum antibiotic prophylaxis and she
delivers a healthy baby girl by vaginal delivery.
Her baby does not require any resuscitation
and remains well in the post natal ward. The
mother is eager for discharge home. What is
the most appropriate course of action with
regards to her child?
Intravenous antibiotics for 24 hours
Check C-Reactive protein levels and take blood
cultures
Discharge if no suspicion of infection
Perform routine 6 hour post natal check and
discharge with community midwife follow up.
Regular observations for 24 hours

A

Maternal colonisation with group B
streptococcus is a minor risk factor for early
onset sepsis in the newborn. Newborns with
only one minor risk factor for early onset sepsis
should remain in hospital for at least 24 hours
with regular observations. Two or more minor
risk factor or one red flag warrant empirical
antibiotic therapy with Benzylpenicillin and
Gentamicin and a full septic screen.

124
Q

A 9-year-old girl is brought to surgery as her
mother is concerned that she is too fat. This
has now been a problem for over two years and
mum feels this is holding her back at school.
What is the most appropriate method to
ascertain how obese she is?
Body mass index
Body mass index percentile adjusted to age and
gender
Weight plotted on percentile chart
Mother’s perception
Waist circumference

A

Defining obesity is more difficult in children
than adults as body mass index (BMI) varies
with age. BMI percentile charts are therefore
needed to make an accurate assessment.
Recent NICE guidelines suggest to use ‘UK 1990
BMI charts to give age- and gender-specific
information’

125
Q

A newborn male baby is found to have an
undescended right testicle during the routine
6-8 week examination. It is neither palpable in
the scrotum or inguinal canal. What is the most
appropriate management?
Outpatient referral to urology to be seen within
4 weeks
Review at 3 months
Immediate referral to urology
Arrange ultrasound abdomen and scrotum
Review at 12 months

A

Undescended testicle - wait 6 months prior to
referral
If the testicle has not descended by around 3
months then referral should be considered for
orchidopexy.

126
Q

A 6-week-old term infant has difficulty feeding
due to increased breathlessness. As the on-call
doctor you are called to review this baby. You
witness the baby feeding and note she is pink
and well perfused but sweating profusely with
and increased respiratory rate. On examination
you hear a soft pan-systolic murmur at the
lower left sternal border.
What is the most likely underlying pathology?
Transient tachypnoea of the newborn
Acute respiratory distress syndrome
Heart failure
Eisenmenger syndrome
Infantile pneumonia

A

Heart failure typically presents in infants with
symptoms of breathlessness worse on exertion
(e.g. feeding), sweating, poor feeding and
recurrent chest infections.
On examination you should: examine the
growth charts (?failure to thrive), examine for
tachycardia, tachypnoea, murmurs and pre and
post-ductal saturations.
Heart failure may be due to duct dependant
systemic circulations (<2 weeks old) e.g.
coarctation of the aorta or left-to-right shunts
(>2 weeks old) e.g. VSD as the pulmonary
vasculature resistance begins to fall.
In this case the baby has a large VSD causing
decompensated heart failure. Cardiac lesions
can be missed during the foetal anomaly scan

and this baby would need a detailed foetal
echocardiogram and discussion with the
cardiac team on management strategies.

127
Q

An 18 month old child attends the paediatric
assessment unit with his mother. He has been
brought in as he has had a fever, barking cough
and difficulty breathing at night. He has been
diagnosed with croup and you have been asked
to see him to review. After history and
assessment you are confident there is no
stridor or respiratory distress. What would your
next step in management be?
Give antibiotics
Give oxygen
Full ENT exam
Give nebulised adrenaline
Give oral dexamethasone

A

This child has mild croup, the severity of croup
is based upon; respiratory rate, respiratory
distress, heart rate, O2 saturations and
exhaustion. Treatment of mild croup is oral
dexamethasone 0.15mg/kg single dose and
review. Systemic dexamethasone and nebulised
adrenaline 5ml of 1:1000 are used in severe
croup, alongside oxygen administration.
Antibiotics should not be given unless an
underlying bacterial infection is suspected. You
should not perform an ENT exam due to the
possibility of an epiglottis diagnosis.

128
Q

A mother notices that her newborn boy has
small eye openings, a small body and low-set
ears. On examination the paediatrician also
notes a flat philtre, a sunken nasal bridge, short
palpebral fissures and a thin upper lip. What is
the most likely cause?
Diabetes
Maternal alcohol abuse
Group B Streptococcal infection
Maternal Listeria
Maternal opioid abuse

A

Fetal alcohol syndrome
Maternal alcohol abuse during pregnancy.
Presentation: IUGR, microcephaly, midfacial
hypoplasia, micrognathia, smooth philtrum,
microphthalmia, short palpebral fissures, thin
upper lip, irritability, ADHD.

129
Q

A 2 day old baby who was born by a ventouse
delivery is noted to have a swelling on the left
side of his head in the parietal region. His head
appeared normal immediately after delivery.
On examination, the baby is well and the
swelling does not cross suture lines. The
fontanelles and sutures appear normal. What is
the most likely diagnosis?
Subaponeurotic haematoma
Caput succedeneum
Craniosynostosis
Skull fracture

Cephalohaematoma

A

A cephalohaematoma appears as a swelling
due to bleeding between the periosteum and
the skull. It is most commonly noted in the
parietal region and is associated with
instrumental deliveries. The swelling usually
appears 2-3 days following delivery and does
not cross suture lines. It gradually resolves over
a number of weeks.

130
Q

A 9-year-old boy who has recently arrived from
India presents with fever. On examination a
grey coating is seen surrounding the tonsils and
there is extensive cervical lymphadenopathy.
What is the most likely diagnosis?
Dengue fever

Typhoid
Paratyphoid
Actinomycosis
Diphtheria

A

Diphtheria

131
Q

A baby born at 35 weeks gestations via normal
vaginal delivery is found to be irritable 48 hours

after birth and suffers a convulsion. There is no
obvious head trauma or swellings. Which one
of the following cranial injuries is most likely to
have occurred?
Caput succedaneum
Cephalohaematoma
Subaponeurotic haemorrhage
Intraventricular haemorrhage
Extradural haemorrhage

A

Caput succedaneum is caused by pressure on
the fetal scalp during the birthing process. It
results in a large oedematous swelling and
bruising over the scalp. Treatment is not
required as the swelling reduces over a few
days.
A cephalohaematoma may occur after a
spontaneous vaginal delivery or following a
trauma from the obstetric forceps or the
ventouse. A haemorrhage results after the
presidium is sheared from the parietal bone.
The tense swelling is limited to the outline of
the bone. It reduces over a few weeks -
months.
A Subaponeurotic haemorrhage, also known as
a subgaleal haemorrhage is rare and is due to a
traumatic birth. It may result in the infant
losing large amounts of blood.
An intracranial haemorrhage refers to
subarachnoid, subdural or intraventricular
haemorrhages. Subarachnoid haemorrhages
are common and may cause irritability and
even convulsions over the first 2 days of life.
Subdural can following the use of forceps.
Intraventricular haemorrhage mostly affects
pre-term infants and can be diagnosed by
ultrasound examinations.
Extradural haemorrhage is unlikely to occur
during the birthing process.

132
Q

You are an FY1 on the paediatric ward round
with your consultant. Whilst seeing a child that
has been admitted with croup, the consultant
you’re with decides to quiz you on the
pathophysiology.
‘What is the most likely organism to cause
croup?’
Respiratory syncytial virus (RSV)
Parainfluenza virus
Pseudomonas aeruginosa
Streptococcus pneumoniae
Bordetella pertussis

A

Parainfluenza

133
Q

A 2-year-old boy presents with a harsh cough
and pyrexia. His symptoms worsened overnight
and on examination stridor is noted. Which one
of the following interventions may improve his
symptoms?
Codeine linctus
Humidified oxygen

Nebulised salbutamol
Oral erythromycin
Oral dexamethasone

A

Oral dexamethasone

134
Q

A 5-year-old girl attends your GP surgery with
her mother. She reports a five day history of a
sore throat and fever. On examination you note
a bright red tongue, flushed face and a rough
dry erythematous rash on her neck.
What is the most likely diagnosis?
Measles
Rubella
Bordetella pertussis
Kawasaki disease
Scarlet fever

A

A strawberry tongue can be seen in both scarlet
fever and Kawasaki disease. However given the
history a diagnosis of scarlet fever is more
likely.

135
Q

An 8-year-old boy presents with weakness and
purple striae on his abdomen. On examination
he is obese with a central fat distribution and is
found to have facial plethora. He is also found
to have a blood pressure of 130/85 mmHg and
facial plethora.
What is the most likely underlying cause?

ACTH-secreting pituitary tumour
Adrenal carcinoma
Craniopharyngioma
Congenital adrenal hyperplasia
Ectopic adrenocorticotropin-producing tumour
A

The history is suggestive of Cushing’s
syndrome. In an 8-year old boy the commonest
cause of Cushing’s would be iatrogenic use of
glucocorticoids. Out of the options above the
most likely answer is an ACTH-secreting
pituitary tumour.

136
Q

589
A 2-year-old child with a history of atopic
eczema is brought to the local GP surgery. Her
eczema is usually well controlled with
emollients but her parents are concerned as
the facial eczema has got significantly worse
overnight. She now has painful clustered
blisters on both cheeks, around her mouth on
her neck. Her temperature is 37.9ºC. What is
the most appropriate management?
Advise paracetamol + emollients and reassure
Admit to hospital
Add hydrocortisone 1%
Oral flucloxacillin
Topical fusidic acid

A

Eczema herpeticum is a serious condition that

requires IV antivirals

137
Q

A baby is born at term via vaginal delivery with
no complications, however he is still not
showing signs of breathing at one minute.
Heart rate is >100bpm, but he is floppy and
cyanosed. What is the most appropriate next
step in management?
Call for anaesthetist to intubate the baby
5 mouth-to-mouth rescue breaths
5 breaths of oxygen via face mask
Start chest compressions
Suction airways

A

Airway suction should not be performed unless
there is obviously thick meconium causing
obstruction, as it can cause reflex bradycardia
in babies. Chest compressions are not
indicated, as the HR in this case is >100bpm.
CPR should only be commenced at a HR <
60bpm. In cases where there are no signs of
breathing and this is thought to be due to fluid
in the lungs, five breaths should be given via a
250ml bag via face mask. This is a more
effective and more hygienic method than using
mouth-to-mouth in a hospital setting.

138
Q

A 14-year-old male being investigated for irondeficiency
anaemia is found to have numerous
polyps in his jejunum. On examination he is
also noted to have pigmented lesions on his
palms and soles. What is the likely diagnosis?
Hereditary non-polyposis colorectal carcinoma
Gardner’s syndrome
Familial adenomatous polyposis
Peutz-Jeghers syndrome
Hereditary haemorrhagic telangiectasia

A

Hereditary haemorrhagic telangiectasia is
associated with mucocutaneous lesions and
iron-deficiency anaemia but intestinal polyps
are not a feature

139
Q

You see a 6 week-old baby boy for his routine
baby check and note a small, soft, umbilical
hernia on examination. What should you do?

Advise parents to tape a coin over the area
Refer for surgery
Refer for ultrasound
Watch and wait
Arrange emergency admission
A

Small umbilical hernias are common in babies
and tend to resolve by 12 months of age.
Parents should be reassured no treatment is
usually required but to be aware of the signs of
obstruction or strangulation such as vomiting,
pain and being unable to push the hernia in -
this is rare in infants. Advise the parents to
present the child at around 2 years of age if the
hernia is still present to arrange referral to a
surgeon. Attempts to treat the hernia by
strapping or taping things over the area are not
helpful and can irritate the skin.

140
Q

A 12-year-old female from Bulgaria presents to
the surgery. She reports being unwell for the
past 2 weeks. Initially she had a sore throat but
she is now experiencing joint pains
intermittently in her knees, hips and ankles. On
examination there are some pink, ring shaped
lesions on the trunk and occasional jerking
movements of the face and hands. What is the
most likely diagnosis?
Lyme disease
Infective endocarditis
Polyarticular juvenile idiopathic arthritis
Rheumatic fever
Still’s disease

A

Rheumatic fever

141
Q

You are reviewing a 9-month-old child with
suspected bronchiolitis. Which one of the
following features should make you consider
other possible diagnoses?
Fine inspiratory crackles
Rhinitis
Feeding difficulties
Temperature of 39.7ºC
Expiratory wheeze

A

A low-grade fever is typical in bronchiolitis.
SIGN guidelines advise that the presence of
high fever should make the clinician carefully
consider other causes before making the
diagnosis.

142
Q

After birth which of the following happens in
the foetus?
The foramen ovale opens allowing blood to
circulate into the pulmonary artery
Haemoglobin A is replaced by Haemoglobin F,
which has a lower affinity for oxygen and may
lead to physiological jaundice in the newborn
The umbilical veins and arteries remain open
for several days
The ductus arteriosus closes
The first few breaths force lung fluid into the
fetal alveoli

A

After birth, the foramen ovale, ductus
arteriosus and umbilical vessels close within a
few hours.
After a few days Haemoglobin F is replaced by
Haemoglobin A, which has a lower affinity for
oxygen and may lead to physiological jaundice
in the newborn, due to the breakdown of fetal
blood cells. The first few breaths force lung
fluid out of the fetal alveoli.

143
Q

An 18-month-old boy is brought to the GP by
his mother as she is concerned about his
breathing. Three days ago he started with fever,
cough and rhinorrhoea. For the past 24 hours
his mother reports that he has been ‘wheezy’.
On examination his temperature is 37.9ºC,
heart rate 126/min, respiratory rate 42/min
and a bilateral expiratory wheeze is noted. You
prescribe a salbutamol inhaler along with a
spacer. Two days later the mother represents
noting the inhaler has made little difference to
the wheeze. Clinical findings are similar,
although his temperature today is 37.4ºC. What
is the most appropriate next step in
management?
Inhaled long-acting beta agonist
Oral prednisolone
Add in regular ipratropium bromide
Oral montelukast or inhaled corticosteroid
Oral amoxicillin

A

This child is likely to have a viral-induced
wheeze, also known as episodic viral wheeze.
First-line treatment is short-acting
bronchodilator therapy. If this is not successful
then either oral montelukast or inhaled
corticosteroids should be tried.

144
Q
What is the most common cause of
hypertension in children?
Renal vascular disease
Congenital adrenal hyperplasia
Renal parenchymal disease
Coarctation of the aorta
Phaeochromocytoma
A

Renal parenchymal disease

145
Q

It is December and you are the paediatric
foundation doctor. A five month old baby is
admitted through the paediatric observation
unit with tachypnoea, tachycardia and fever. On
examination there is evidence of increased
work of breathing with sub costal and
diaphragmatic recession. There is widespread
wheeze. You discuss the patient with your
senior and a diagnosis of bronchiolitis is
established. Oxygen is started but it is not
deemed appropriate to begin intravenous fluids
at this time. Which investigation is important to
conduct in the management of this patient?
Glucose
Full blood count
Nasopharyngeal aspirate
Urea and electrolytes
Arterial blood gas

A

Nasopharyngeal aspirate are recommended
during the winter months to ascertain which
children are suffering with respiratory syncitial
virus positive bronchiolitis. By diagnosing these
patients it helps with ward management of
patients, placing RSV negative patients on a
ward and RSV positive patients in a side room.
Urea and electrolytes would only be
appropriate if the patient was on IV fluids.

146
Q

Charlie is a 7 month old baby boy who presents
to you with poor weight gain (50th to 10th
centile), on examination he has an
erythematous, blanching rash over his
abdomen, colicky abdominal pain and vomiting
after feeds. He has been breast feeding with
top ups of ‘Aptamil’ formula. What is the most
likely diagnosis?
Pyloric stenosis
Eczema
Infantile colic
Cows’ milk protein intolerance
Reflux

A

The correct answer is cows’ milk protein
intolerance.
The following clues in the history would suggest
the diagnosis of cows’ milk protein intolerance:
Multi-system involvement
7 months would suggest the new introduction
of top up feeds which correlates with the
symptoms
Faltering growth along with the multi-system
involvement would suggest cows’ milk protein
intolerance
Charlie is older than the classical age of
presentation for pyloric stenosis (2 to 8 weeks

very rare above 6 months)
The presentation is unusual for eczema,
infantile colic and reflux due to the multisystem
involvement in the history making cows'
milk protein intolerance more likely.
147
Q

A newborn is found to have a number of
congenital abnormalities including an extra
finger on each hand, a cleft palate and lip,
microphthalmia and microcephaly.
Which of the following chromosomes is most
likely to be affected in this child?
9
12
13
18
21

A

Patau syndrome is a chromosomal abnormality
resulting in an extra full copy of chromosome
13 (trisomy 13). Like many of the chromosomal
defects, physical and mental disability is
common, in this case key distinguishing
features to separate Patau’s from other trisomy
disorders include polydactyly, cleft lips and
palates, microcephaly and microphthalmia.
Many children die before within a year of birth
but those who survive will often go on to show
intellectual and motor disability.

148
Q
14-year-old boy is brought in by his mother
who noticed her child had repeated episodes of
slurred speech and gait abnormalities. On
musculoskeletal examination, you notice
muscle weakness, dysdiadochokinesis and
spinal scoliosis. What is the mode of
inheritance of this condition?
X-linked recessive
Autosomal dominant
Point mutation
X-linked dominant
Autosomal recessive
A

Firedrich’s ataxia

AR

149
Q
A 6-year-old boy is diagnosed as having
nephrotic syndrome. A presumptive diagnosis
of minimal change glomerulonephritis is made.
What is the most appropriate treatment?
Cyclophosphamide
Albumin infusion
Plasma exchange
Renal biopsy followed by prednisolone
Prednisolone
A

A renal biopsy is only indicated if response to

steroids is poor

150
Q

A 29-week-old baby is born premature and
shortly after birth experiences tachypnoea and
tachycardia along with chest wall retractions.
The paediatrician notes that the neonate has a
blue discolouration of the skin and commences
continuous positive airway pressure (CPAP) and

intravenous fluids before explaining to the
parents that the lungs lack surfactant, a
compound that helps people breathe.
Which of the following cells are responsible for
surfactant production?
Microfold cells
Alveolar macrophage
Type 1 pneumocytes
Type 2 pneumocytes
Paneth cells

A

Type 1 pneuomcytes are involved in the
process of gas exchange between the alveoli
and the blood and type 2 pneumocytes
produce pulmonary surfactant

151
Q

A mother brings her son in to surgery as she
suspects he has a squint. She thinks his right
eye is ‘turned inwards’. You perform a cover
test to gather further information. Which one
of the following findings would be consistent
with a right esotropia?
On covering the left eye the right eye moves
medially to take up fixation
The cover test could not be used to identify this
type of defect
On covering the left eye the right eye moves
laterally to take up fixation

On covering the right eye the left eye moves
laterally to take up fixation
On covering the right eye the left eye moves
medially to take up fixation

A

On covering the left eye in this example the
right eye moves laterally from the nasal
(esotropic) position to take up fixation.

152
Q

A neonate who was born prematurely at 35
weeks gestation is registered at the Practice. He
was very well after delivery, without any
notable complications such as respiratory
problems. How should his routine childhood
immunisations be given?
Adjust schedule for gestational age
Give according to chronological age
Refer to the hospital to receive first
immunisations
Start immunisations at 3 months old
Delay until weight reaches 3.5kg

A

Babies who were born prematurely should
receive their routine vaccinations according to
chronological age; there should be no
correcting for gestational age. Babies who were
born prior to 28 weeks gestation should receive
their first set of immunisations at hospital due
to risk of apnoea.

153
Q

A 14-year-old attends surgery. She was
diagnosed with having migraines three years
ago and requests advice about options for
treating an acute attack. Which one of the
following medications is it least suitable to
recommend?
Aspirin
Paracetamol + prochlorperazine
Paracetamol + codeine
Ibuprofen

Paracetamol

A

Avoid aspirin in children < 16 years as risk of
Reye’s syndrome
Aspirin should be avoided in children due to
the risk of Reye’s syndrome.
Codeine would also be a poor choice as it has
limited benefit in migraine.

154
Q

You are reviewing a 11-month-old baby with a
viral upper respiratory tract infection. She is
clinically well but at the end of the consultation
her mother asks you about her development.
You notice that she points and babbles ‘mama’
and ‘dada’ but has no other words. She is shy
and cries when you try to examine her. There is
an early pincer grip and she can roll from front
to back but she cannot yet sit without support.
How would you describe her development?
Normal development
Global developmental delay
Isolated delay in gross motor skills
Delay in speech + social skills, possibly early
autism
Isolated delay in fine motor skills

A

Most babies can sit without support at 7-8
months so this probably represents a delay in
gross motor skills. If still present at 12 months
she should be considered for referral to a
paediatrician. The other development features
are normal for her age.

155
Q

Meera brings her 5 year old daughter Reena to
the surgery who is being treated for acute
lymphoblastic leukaemia (ALL) for review as
Reena’s classmate has been sent home from
school with chickenpox. Reena is asymptomatic
currently. Meera is unclear if Reena has
suffered with chicken pox previously. What
would be the correct management?
Admit urgently
Send home and come back if symptomatic
Prescribe aciclovir
Prescribe varicella zoster immunoglobulin
Urgent bloods for varicella zoster antibodies

A

‘People who have had a significant exposure to
chickenpox and who are immunocompromised
should be tested for varicella-zoster antibody,
regardless of their history of chickenpox. Test
for varicella-zoster immunoglobulin G (IgG)
antibodies in primary care if test results can be
available within 2 working days of first
exposure. If this is not possible, urgently seek
specialist advice because testing in secondary
care and/or varicella-zoster immunoglobulin
prophylaxis may be needed.’

156
Q

A newborn baby has their blood glucose
measured on the post natal ward as part of the
neonatal hypoglycaemia protocol due to low
birthweight. It measures 2.9mmol/L. The
midwife asks you what you want to do next?
Admit to the Special Care Baby Unit (SCBU) for
NG feeding
Offer additional feed if willing
Administer 100 mls intravenous 20% glucose
Measure blood glucose again in three hours
time
Take blood sample for a formal glucose
measurement

A

Neonatal hypoglycaemia is a common medical
problem affecting neonates. This usually
represents adaption to extrauterine life as
opposed to any significant underlying medical
problems. In the neonate blood glucose levels
of >2.5mmol/L are usually regarded as normal.
Formal measurements may be needed to
confirm readings of either extreme as they are
more reliable. If measurements are consistently
>2.5mmol/L then monitoring can be stopped.

157
Q

Which one of the following statements
regarding absence seizures is incorrect?
Typical age of onset of 3-10 years old
Sodium valproate and ethosuximide are firstline
treatments
Seizures may be provoked by a child holding
their breath
There is a good prognosis
The EEG characteristically shows a bilateral,
symmetrical 3Hz spike and wave pattern

A

Seizures are characteristically provoked by

hyperventilation

158
Q
A 4-year-old boy is admitted after developing a
haemarthrosis in his right knee whilst playing in
the garden. The following blood results are
obtained:
Platelets220 * 109/l
PT12 secs
APTT78 secs
Factor VIIIc
activityNormal
What is the most likely diagnosis?
Antithrombin III deficiency
Von Willebrand's disease
Antiphospholipid syndrome
Haemophilia A

Haemophilia B

A

A grossly elevated APTT may be caused by
heparin therapy, haemophilia or
antiphospholipid syndrome. A normal factor
VIIIc activity points to a diagnosis of
haemophilia B (lack of factor IX).
Antiphospholipid syndrome is a prothrombotic
condition

159
Q

6-week-old infant is referred from the health
visitor due to failure to thrive. The infant has
fallen from the 50th to 9th centile on growth
chart for weight. On further questioning, the
parents reveal the infant vomits following each
meal, which have on occasions ‘hit the wall.’
The mother’s pregnancy was unremarkable,
with normal antenatal scans, and the infant
was born by an uncomplicated vaginal delivery.
There were no abnormal features noted at the
newborn baby examination. What is the most
likely diagnosis?
Intussusception
Infantile colic
Cow’s milk protein intolerance
Galactosaemia
Pyloric stenosis

A

In this question the most likely diagnosis is
pyloric stenosis. Pyloric stenosis typically
presents around 2-6 weeks of age. Infants tend
to have projectile vomiting following feeds and
remain hungry after vomiting. There may be an
olive shaped mass in the right upper quadrant
due to hypertrophy of the pylorus, and ‘waves
of peristalsis’ may be seen following a test
feed.

160
Q

A man brings his 18 month old daughter to
your GP clinic. She has had coryzal symptoms
for the last 2 days. Last night, she started with a
barking cough and a mild temperature of
37.8º.
On examination, there is a mild stridor when
mobilising, with no recessions visible. Chest
sounds clear with good air entry bilaterally.
Temperature today remains at 37.8º, but all
other observations are normal. What is the
appropriate management?
Admit to hospital
Give nebulised adrenaline
Give a stat dose of dexamethasone 150
micrograms/kg PO
Give a salbutamol inhaler
Start antibiotics

A

This is a child who has croup. This is an illness
that usually starts with coryzal symptoms, and
the child then develops a seal like, barking
cough.
The first stage is to work out how serious a case
of croup this child has. Generally
recommendations include:
Mild croup:
Occasional barking cough with no stridor at
rest
No or mild recessions
Well looking child
Moderate croup:

Frequent barking cough and stridor at rest
Recessions at rest
No distress
Severe croup:
Prominent inspiratory stridor at rest
Marked recessions
Distress, agitation or lethargy
Tachycardia
In this case, the child would have mild croup.
Admission to hospital is only considered for
moderate or severe croup, or if an alternative
severe diagnosis like epiglottitis is suspected. It
would not be appropriate in this case.
Nebulised adrenaline would only be used for
children who were distressed, or who had a
severe stridor. It would be not be used in this
case as this child is well at rest with only a mild
stridor on movement.
A salbutamol inhaler would only help if the
child had wheeze, which she does not in this
case. It would not give her any benefit.
Antibiotics are not indicated in croup as it is a
viral illness.
Systematic reviews have shown that steroids
can ease symptoms within a few hours. They
also lead to fewer reattendances and fewer
hospital admissions. Mild croup will resolve on
its own, but Dexamethasone has been shown
to be of some benefit.

161
Q

BB. A 5-year-old girl is brought to A+E after school by her mother and school teacher with central abdominal pain.

The best history is likely to be obtained by talking to:

Answers:
a.
Her mother

b.
Her mother and her school teacher

c.
Her school teacher

d.
The A+E triage nurse

e.
girl herself and her mother

A

girl herself and mother

162
Q

BB. A two-year-old boy is seen in Paediatric Out-Patients with a 6 month history of diarrhoea and parental concern about his nutritional status. The best way to assess these concerns would be to:

a.
Assess his weight for height

b.
Calculate his Body Mass Index (BMI)

c.
Measure his height and weight in clinic

d.
Measure his upper arm circumference
e
Review and plot serial weights and heights

A

e

Review and plot serial weights and heights

163
Q

BB

A previously healthy 2-year-old child presents with a 24 hour history of diarrhoea and vomiting. Which of the following is the single, most accurate method for assessing the degree of dehydration?

a.
Assess skin turgor

b.
Assess the fontanelle

c.
Calculate the difference between the current weight and the predicted weight from the child’s growth records

d.
Examine the mucous membranes

e.
Measure the heart rate and blood pressu

A

c.

Calculate the difference between the current weight and the predicted weight from the child’s growth records

164
Q

BB. A 3-year-old boy attends the Paediatric A+E Department because he has developed an itchy rash whilst at a birthday party. Of the following features, which requires immediate treatment with 0.01 ml/kg of 1:1000 adrenaline i.m.?

a.
Blood pressure of 88/50

b.
Generalised urticaria

c.
Lip swelling

d.
Respiratory rate of 22/minute

e.
Wheeze on auscultatio

A

e.

Wheeze on auscultatio

165
Q

BB
A 3-year-old boy has just started fitting in A+E. You are the F1 doctor. The nurses are all busy with other children. Of the following actions which should you do first?:

a.
Administer oral midazolam

b.
Do a blood gas

c.
Gain iv access

d.
Give him high-flow oxygen

e.
Measure his blood sugar

A

d.

Give him high-flow oxygen

166
Q

BB
A 15-month-old girl presents with a 3-day history of intermittent fevers and vomiting and poor feeding. On examination her temperature is 38.6 C but there are no localising signs. What is the most likely cause?

a.
Bronchiolitis

b.
Encephalitis

c.
Meningitis

d.
Pyloric stenosis

e.
Urinary tract infection

A

UTI

167
Q

BB
A three year old girl presents with a two day history of fever of 37.8º and inspiratory stridor. She has a loud barking cough which is worse in the night. What is the most likely diagnosis?

a.
Acute exacerbation of asthma

b.
Bacterial tracheitis

c.
Croup

d.
Epiglottitis

e.
Pneumonia

A

Croup

168
Q

BB
A term neonate is cyanotic and tachypnoeic at birth. There is soft pan systolic murmur at the lower sternal edge. An antenatal scan had shown a VSD, overriding aorta and infundibular stenosis. What is the most likely diagnosis?

a.
Ebstein anomaly

b.
Tetralogy of Fallot

c.
Total anomalous pulmonary venous return

d.
Transposition of the great arteries

e.
Truncus arteriosus

A

b.

Tetralogy of Fallot

169
Q

BB
A 6 yr old male born in the UK to Iraqi parents, develops sudden onset very dark red urine with no dysuria. He has a intercurrent viral infection. On exam, slightly jaundiced, no liver or spleen; urine – blood ++++. There was a history of prolonged neonatal jaundice. Which investigation is most likely to reveal the underlying diagnosis?

a.
Blood film

b.
G6PD level, now and in one month’s time

c.
Hb electrophoresis

d.
Liver function tests

e.
Urine M,C&S

A

b.

G6PD level, now and in one month’s time

170
Q

BB
A twelve year old boy has had seven episodes of spontaneous lip swelling and bilateral periorbital oedema in the last three years. His father also had similar episodes in childhood. What is the most likely immunological mediator?

a.
C1 esterase inhibitor

b.
Complement C4

c.
Eosinophils

d.
Histamine

e.
IgE antibodies

A

a.

C1 esterase inhibitor

171
Q

BB
One evening a 4-month-old boy is brought into A+E by his step-father because of a prolonged nose bleed. This has now stopped. The examination is normal apart from some irregular bruising on his abdomen and small, well circumscribed marks on his back. The next most appropriate step from the options below would be to:

a.
Admit him and arrange a full skeletal survey, clinical photographs and clotting studies

b.
Check his full blood count and clotting studies, and discharge him if they are normal for follow up in OPD

c.
Discharge him and discuss the case the next day with the child protection team

d.
Discuss the case with the child protection team now

e.
Reassure the step-father that since the nose bleed has stopped, no further treatment is required

A

d.

Discuss the case with the child protection team now

172
Q

BB
An eight year old boy presents with nocturnal cough and early morning tiredness. He has a past history of serous otitis media. On examination, he has noisy breathing, is overweight and is inattentive during consultation. What is the most likely diagnosis?

a.
Asthma

b.
Chronic bronchitis

c.
Obstructive sleep apnoea

d.
Pertussis

e.
Pulmonary tuberculosis

A

c.

Obstructive sleep apnoea

173
Q

BB
A term baby is born normally at 38 weeks gestation at 2.3 kg. Labour was difficult with prolonged rupture of membranes. At 35 minutes of age the infant was noted to have an increasing oxygen requirement with grunting and respiratory distress. On examination he was floppy with an oxygen saturation of 95% in 2L/min of oxygen. A CXR showed reticulonodular shadowing. What is the most likely diagnosis?

a.
Bacterial pneumonia

b.
Bronchiolitis

c.
Chronic lung disease

d.
Persistent pulmonary hypertension

e.
Respiratory distress syndrome

A

a.

Bacterial pneumonia

174
Q

BB
A 2-month-old, ex-26 week gestation boy establishing feeding on the neonatal unit has developed a distended abdomen and bile-stained vomiting, and has an increasing oxygen requirement. The most likely diagnosis is:

a.
Appendicitis

b.
Gastro-oesophageal reflux

c.
Hirschsprung’s disease

d.
Intussusception
e.
Necrotising enterocolitis

A

e.

Necrotising enterocolitis

175
Q

An 18 month old boy is seen with a history of recurrent oral candidiasis. He developed tetany in the newborn period and had cardiac surgery for a complex heart defect. What is the most likely immunological deficiency?

a.
B-cells

b.
Complement C3

c.
Neutrophils

d.
NK cells

e.
T-cells

A

e.
T-cells

T cells - de George’s syndrome.

Tetany - hypocalcaemia, parathyroid deficient, absence of thymus.

176
Q

A 5-year-old girl who never received MMR was exposed to chicken pox last week at school. She now presents with a 12 hour history of rash and abdominal pain and is reluctant to walk. On examination, she is unwell with cold hands and feet, and a widespread blanching maculopapular rash with spots of different sizes sparing the head and neck. What is the most likely diagnosis?

a. 
Chicken pox (Varicella zoster)

b.
Measles

c.
Meningococcal sepsis

d.
Roseola infantum

e.
Rubella

A

c.
Meningococcal sepsis

Rash doesn’t have to be non blanching.

Cold hands and feet - think seriously about sepsis.

Rosella - fever, fever, fever, rash comes after.

177
Q

BB
A ten-year-old girl is seen in the endocrine clinic for short stature. On examination she is noted to have widely spaced nipples, a webbed neck, cubitus valgus and an ejection systolic murmur at the upper sternal edge. What is the most likely diagnosis?

a.
Aortic valve anomaly

b.
Innocent murmur

c.
Patent ductus arteriosus

d.
Pulmonary stenosis

e.
Tetralogy of Fallot

A

a.
Aortic valve anomaly
- coarctation of the aorta…

Outflow Obs in Well Child Adult – Type Coarctation of Aorta
Not duct dependent – more severe over years.
- Asymptomatic.
- Systemic HT in R arm.
- ESM at upper sternal edge.
- Collaterals heard – continuous murmur at back.
- Radio-femoral delay.
CXR – rib notching – collateral intercostal arteries. 3 sign – visible notch, in descending aorta at site coarctation.
ECG – LVH.
Rx - Stent

Outflow Obs in Unwell Child Coarctation of Aorta
Arterial duct tissue encircling the aorta, at point of insertion of duct.
Duct closes – aorta constricts = severe obstruction to LV outflow.

Circulatory collapse – 2 days.
-	HF.
-	Absent femoral.
-	Severe metabolic acidosis.
CXR – cardiomegaly
ECG – normal.
Rx – surgery. Prostaglandin infusion.
178
Q

BB
A 5-month-old boy has been brought to A+E 2 hours previously with a 12 hour history of very high fever and vomiting. Examination does not reveal a clear source of infection so a blood culture is taken and lumbar puncture is performed. Of the following options, which method reflects current clinical practice when an urgent urine sample is required prior to starting antibiotics?

a.
Fixing a sterile bag and waiting for a sample

b.
Performing an in-out urinary catheterisation with aseptic technique

c.
Placing a sterile pad in the nappy and waiting for a sample

d.
Placing and leaving a urinary catheter in situ

e.
Waiting for the parents to ‘catch’ a mid-stream urine sample in a bowl

A

b.

Performing an in-out urinary catheterisation with aseptic technique

179
Q

BB
A four year old, known epileptic is admitted to the ward for observation as he has been vomiting. He is on regular sodium valproate. You are called urgently to the ward to see him. An hour ago he had his regular sodium valproate but vomited soon afterwards. He is now having a seizure and has been fitting for about six minutes. A fingerprick blood glucose is 4.7 mmol/l. What is the most appropriate initial action?

Answers:
a.
Give an anti-emetic

b.
Give buccal midazolam

c.
Give IV dextrose

d.
Give IV phenytoin

e.
Start a sodium valproate infusion

A

b.

Give buccal midazolam

180
Q

BB
A 3 year old girl’s parents have noted that her urine has gone very dark and she has had puffy eyes. She has been unwell recently with a fever and sore throat. Urine shows blood ++++, protein +++. BP 110/65. Which investigation is most likely to indicate the underlying diagnosis?

a.
24-hour urine protein quantification

b.
Blood culture

c.
Complement C3 and C4 levels

d.
Ultrasound scan of the abdomen

e.
Urine M,C&S

A

c.
Complement C3 and C4 levels

Post strep glomerulonephritis - complement C3 and C4 levels - indicate underlying diagnosis.

181
Q

BB
A 4-month-old girl is brought into A+E Resus by paramedics with a widespread, progressing, non-blanching purpuric rash on both lower limbs. She has cold peripheries with a capillary refill time of 5 seconds, has a heart rate of 180/min and her blood pressure is unrecordable. The most likely diagnosis is:

a.
Anaphylaxis

b.
Erythema infectiosum

c.
Group B streptococcal infection

d.
Henoch-Schonlein purpura (HSP)
e.
Neisseria meningitidis infection

A

e.

Neisseria meningitidis infection

182
Q

BB
A 3-year-old child presents with acute onset abdominal pain for 4 hours. She has
had 3 similar episodes in the past, which were all self-resolving. This time the
vomiting is bile-stained. What is the most likely cause?

a. Food allergy
b. Intussusception
c. Malrotation
d. Pyloric stenosis
e. Strangulated inguinal hernia

A

c. Malrotation

183
Q

BB
A 5 year old who has recently returned from Nigeria, has had 3 days of swinging
high fevers. His urine is noted to be dark red. On examination he has an enlarged
liver, but no other abnormal ndings.
A urine dipstick is positive for blood +++.
Which investigation is most likely to reveal the underlying diagnosis?

a. Blood culture
b. G6PD level, now and in one month’s time
c. Hb electrophoresis
d. Thick and thin blood lm
e. Urine M,C&S

A

d. Thick and thin blood film

184
Q

BB

A 2-year-old boy presented to A+E with a 4 week history of intermittent high fever
and misery. The parents have noticed a widespread evanescent salmon-pink rash. In
the last 48 hours he has been unwilling to weight bear. Laboratory tests reveal
markedly raised acute phase markers. The most likely diagnosis is:

Answers: a. Chronic listeria infection
b. Dermatomyositis
c Duchenne muscular dystrophy
c. Duchenne muscular dystrophy
d. Juvenile idiopathic arthritis
e. Systemic lupus erythematosus
A

d. Juvenile idiopathic arthritis

185
Q

BB
A 7 year old girl with severe wheeze, a peak expiratory flow
rate of 40%and a heart
rate of 120 bpm presented to A+E an hour ago. She has been treated withburst
therapy using Salbutamol and Ipratroprium Bromide but has had no response. What
is the next most appropriate treatment

a. Adrenaline (epinephrine) subcutaneously
b. Aminophylline infusion
c. Magnesium sulphate orally
d. Prednisolone orally
e. Salbutamol iv

A

e. Salbutamol iv

186
Q

BB
A 14-month-old boy, with normal growth is referred to you due to his mother
complaining that he is a dicult
feeder. He refuses most solids and meal times are a
battle, but he likes drinking cows’ milk and has about 1 litre per day. Which of the
following would you do next?

a. No need to test for deciency
in the absence of clinical signs
b. Test for calcium deciency
c. Test for folic acid deciency
d. Test for iron deciency
e. Test for vitamin B12 deciency
A

d. Test for iron deciency

187
Q

BB

A 14-year-old girl presents with a 5 day history of abdominal pain, submandibular
and parotid gland enlargement, low grade fever and severe headaches. She has not
received any antibiotics. On examination she is photophobic with neck stiness,
but
is well perfused. She has no rash. On lumbar puncture the cerebrospinal fluid
has
normal protein and glucose, 50 WBC/ml (95% lymphocytes) and no RBC. What is the
most likely diagnosis?

a. Bacterial meningitis
b. HIV infection
c. Measles
d. Mumps
e. Tuberculosis

A

Mumps

188
Q

BB

A 15-month-old Muslim girl is not meeting her gross motor developmental
milestones and is generally very irritable. She was growing onthe 50th centile, but is
now on the 25th. She has2-hourly breast feeds and consumes minimal solids with
no additional dairy products. What is the most likely diagnosis?

a. Failure to thrive
b. Iron Deciency
c. Nutritional Rickets
d. Phosphate deciency
e. Vitamin A deciency

A

c. Nutritional Rickets

should be eating…

15 month old - milk, in morn, eve. 3 meals a day plus snacks.

189
Q

BB
A 6 month old is examined by the GP because of parental concern about abdominal
distension over the last 6 weeks. On examination he is well and thriving and has a
palpable right sided abdominal mass. He has microscopic haematuria. Which
investigation is most likely to reveal the underlying diagnosis?

a. Blood lm
b. Clotting studies
c. Ultrasound scan of the abdomen
d. Urine M,C&S
e. Urine protein:Creatinine ratio

A

. Ultrasound scan of the abdomen

190
Q

BB

A 15 year old girl attends the Emergency Department with her boyfriend, also 15,
requesting the morning-after pill following a condom accident the previous
evening.She reveals that four months ago she was circumcised during a family trip
to Somalia. She understands your advice and the implications of her decisions to
engage in sexual activity. She refuses to inform her parents. The couple are using
condoms regularly. What is the appropriate management?

a.
Decline to prescribe the morning-after pill and inform her parents so
that they can support her
b.
Decline to prescribe the morning-after pill and refer the patient back
to her GP
c.
Prescribe the morning-after pill and immediately alert the
safeguarding children’s team
d.
Prescribe the morning-after pill and recommend that the girl informs
her parents
e.
Prescribe the morning-after pill, give contraceptive advice and inform
her parents

A

c.
Prescribe the morning-after pill and immediately alert the
safeguarding children’s team

191
Q

A 7-month-old girl adopted from an orphanage in Nepal is below 0.4th centile for
length and weight. The birth history was apparently normal, apart from prolonged
jaundice postnatally. She is not yet sitting. On examination she is hypotonic and
accid.
The lower limb reexes
are hard to elicit. What is the most likely underlying
condition?

a. Cerebral palsy
b. Congenital hypothyroidism
c. Down syndrome
d. Muscular dystrophy
e. Myasthenia gravis

A

b. Congenital hypothyroidism

192
Q

BB
Athree year old girl is brought into A+E by her parents. She is usually fit
and well.
Her mum says that half an hour ago she ran into a table and banged her head on
the corner. She went pale and fell to the floor,
and then had two or three twitching
movements before starting to cry. She is now back to her normal self and is running
round A+E. Clinical examination is normal. What is the most appropriate action?

a. Organise an ECG
b. Organise an EEG
c. Organise neuro-imaging
d. Reassure the parents
e. Teach parents how to give buccal midazolam if this recurs

A

d. Reassure the parents

193
Q

BB
A 6-year-old caucasian boy has a three-week history of fevers, not responding to oral
antibiotics in the first
and second weeks, increasing drowsiness and now presents
with confusion. On examination he has a mildly stiff
neck and his Glasgow Coma
Score is 12. A lumbar puncture revealed raised protein (2.1 g/l.) in the cerebrospinal
fluid,
decreased glucose, 710 WBC/ml, no RBC and no organisms seen on Gram
stain. What is the most likely underlying cause?

a. Bacterial meningitis
b. HSV encephalitis
c. Meningococcal sepsis
d. Toxoplasmosis
e. Tuberculosis

A

e. Tuberculosis

WBC = increased. normal 0.5mm^3
normal protein = 0.15-0.4 g/L

TB meningitis.

Bcg - very good against TB meningitis.

194
Q

BB
An 8-year-old girl is referred to outpatient department with a 3-month history of
acne, breast development and ne
pubic hair. She is otherwise healthy and has had
no known illnesses, nor is she on any long term medication. Examination is
otherwise normal. What is the most likely aetiology?

a. Afeminising ovarian tumour
b. Agonadotropin producing tumour
c. Congenital adrenal hyperplasia, late onset
d. Early onset of normal puberty
e. Premature adrenarche

A

d. Early onset of normal puberty

195
Q

BB
A 2-year-old boy from a socially disadvantaged family presents with dental caries
and frequent upper respiratory tract infections. He is a very fussy eater. On
examination he has chronic suppurative otitis media but is avidly sucking from a
bottle of cow’s milk. He is continuing to grow along the 25th centile for height and
weight. What is the most likely underlying condition?
a. Acquired immune deciency
(AIDS)
b. Cystic brosis
c. IgA deciency
d. Iron deciency
anaemia
e. Vitamin D deciency

A

d. Iron deciency

anaemia

196
Q

BB
A 15 year-old girl was brought to the A&E after disclosing to her teacher at school
that she had taken about 40 tablets of paracetamol the evening before. On arrival
she was alert but complaining of nausea and right upper quadrant abdominal pain.
Bloods taken on arrival at A&E showed a raised ALT and AST and abnormal
coagulation studies. What is the most appropriate treatment?
a. Activated charcoal
b. Desferrioxamine
c. Gastric lavage
d. N-acetylcysteine
e. No treatment - admit for observation

A

d. N-acetylcysteine

197
Q

BB
A 3-year-old boy with diarrhoea and vomiting was given metoclopromide and
loperamide. He presents to A&E with torticollis and an intermittent convergent
squint and upward deviation of both eyes. What is the most appropriate treatment?
a. Atropine
b. Bromocriptine
c. Buccal midazolam
d. No treatment - admit for observation
e. Procyclidine

A

e. Procyclidine
- anticholinergic drug, used to treat drug0induced parkinsonism, akathisia, acute dystonia, parkinsons.

Dystopia reaction.

Procycline.

198
Q

BB
A 13-year-old boy presents to his GP for a routine physical prior to participation in
competitive sports. He has had no recent illnesses and no past medical history of
note. His height and weight are on the 75th centile for his age. Cardiovascular
examination reveals a grade 2/6 ejection systolic murmur heard loudest at the left
lower sternal border. It is low pitched and musical but does not radiate. Which of the
following is the most likely diagnosis?

a. Atrial septal defect
b. Mitral stenosis
c. Tricuspid regurgitation
d. Ventricular septal defect
e. Vibratory innocent murmur

A

e. Vibratory innocent murmur

199
Q

BB
A seven year old girl who has previously been fit
and well presents in A+E following a
left-sided seizure that lasted approximately three minutes at school. Over the last
few weeks she has been complaining of headaches. She has not had any fever.
What is the most appropriate next step in her management?
a. Do a lumbar puncture
b. Organise and EEG
c. Organise neuro-imaging
d. Start IV acyclovir
e. Start IV ceftriaxone

A

c. Organise neuro-imaging

200
Q

BB
A five
week old girl presents with a history of recurrent coughing and choking during
and after feeds. Pregnancy was complicated by polyhydramnios. What is the most
likely diagnosis?

Answers: a. Epiglottis

b. Gastro-oesophageal reux
c. Laryngomalacia
d. Tongue-tie
e. Tracheo-oesophageal fistula

A

e. Tracheo-oesophageal fistula

201
Q

BB
An 8-week-old exclusively breastfed baby boy was born at 38 weeks gestation after
an uneventful pregnancy. Which of the following findings
would require a prompt
referral to a paediatric surgeon?

a.
Aright testis which can be manipulated to the base of the scrotal
sac
b. An acute episode of balanitis
c.
An irreducible firm
lump which extends from the inguinal canal to
the scrotum
d. Bilateral nontender scrotal swellings which transilluminate
e. Glandular hypospadias

A

An irreducible firm
lump which extends from the inguinal canal to
the scrotum

202
Q
BB
An infant of 31 weeks gestation was born via an emergency section because of
foetal decelerations noted on CTG. The birth weight was 1100g and the infant
required intubation but was difficult
to ventilate. A CXR shows diuse
whiteout of
both lungs with air bronchograms. What is the most likely diagnosis?
a. Bacterial pneumonia
b. Chronic lung disease
c. Meconium aspiration
d. Respiratory distress syndrome
e. Transient tachypnoea of the newborn
A

d. Respiratory distress syndrome

203
Q

BB
A healthy 4-year-old girl develops acute onset of petechiae and epistaxis. Laboratory
ndings
include haemoglobin = 12g/dL; white cell count = 5.5 with normal
dierential;
and platelet count = 15. Of the following investigations, which would you
do next?

a. Blood film
b. Bone marrow aspirate
c. Check the bleeding time
d. Clotting studies
e. Platelet antibody tests

A

a. Blood film

ITP - most likely. Platelets - whole fillers for capillaries, them fibrin cascades and fixes them.

Not bleeding time - wouldn’t cut a child.

Haemophilia - haemarthrosis is main worrying.

Blood film - make sure it’s not leukaemia.

204
Q
BB
A 6-year-old South African boy, domiciled in the UK, has just returned from visiting
his family in Cape Town during July and August. He has a high fever with rigors, is
coughing and is complaining of muscular aches. What is the most likely cause of this
infection?
a. Dengue
b. Influenza
virus
c. Malaria
d Meningococcal sepsis
d. Meningococcal sepsis
e. Tuberculosis
A

b. Influenza

virus

205
Q
BB
A four month old boy who is well and thriving has blood test results positive for HIV
antibodies. What is the most likely immunological mediator of the positive result in
this child?
a. B lymphocytes
b. CD4+ T lymphocytes
c. IgG antibodies
d. NK cells
e. Tranplacental maternal lymphocytes
A

c. IgG antibodies

206
Q

BB
A 10-month-old girl has grown along the 50th centile for weight until around 6
months but is now on the 25th centile. The mother has noticed a distended
abdomen and says her stools are more frequent and bulky. What is the most likely
underlying condition?

a. Coeliac disease
b. Congenital hypothyroidism
c. Cystic brosis
d. Giardia lamblia infection
e. Toddler’s diarrhoea

A

a. Coeliac disease

you start weaning at 6 months - thats why her centile dropped then

207
Q

BB
A 2-week-old boy is brought to A+E by his mother who has noticed that he has
become increasingly jaundiced. She reported that he had not been breast-feeding as
well as previously, and also commented that his stools looked like “off-white
chewing gum”. The most likely diagnosis is
a. Breast-milk jaundice
b. Congenital gall stones
c. Extra-hepatic biliary atresia
d. Glucose 6-phosphate dehydrogenase deciency
e Rhesus incompatibility

A

c. Extra-hepatic biliary atresia

208
Q

BB
The laboratory calls you to inform you that a baby who is now 10 days old has an
elevated TSH on their Guthrie Card test. If this condition is left untreated, which of
the following signs is the baby most likely to demonstrate in the first
few months of
life?

a. Diarrhoea
b. Hyperirritability
c. Hyperphagia
d. Hyperreexia
e. Prolonged jaundice

A

e. Prolonged jaundice

209
Q

BB
A three month old baby has been admitted to the ward with a history of fever and a
two minute generalised, self-terminating tonic-clonic seizure. She has had a lumbar
puncture that shows white blood count 850 per mm3 (80% polymorphs), protein
1.2g/l, glucose 1.7 mmol/l (blood glucose 5.1 mmol/l). What would be the most
appropriate next step in her management?
a. Organise and EEG
b. Organise neuro-imaging
c. Request a paediatric neurology opinion
d. Start IV aciclovir
e. Start IV ceftriaxone

A

e. Start IV ceftriaxone

WBC = very raised
n = 0.5
viral = lymphocytes raised (inital may be polymorphs)
bac = polymorphs, very raised 

protein is very raised (N - 0.14-0.4)
bac = vr,
viral n/r

glucose (n - >50% of blood)
bac = vl
viral = n/l

210
Q

BB
A 2-week-old boy is admitted with a 3-day history of vomiting and increasing
lethargy. Physical examination is normal except for increased pigmentation of the
areolar and nipples bilaterally. Laboratory ndings
include plasma sodium = 126
mmol/L; plasma potassium = 6.8 mmol/L; and plasma glucose = 5.9 mmol/ L. What is
the most likely diagnosis?

a. Congenital adrenal hyperplasia
b. Gastroenteritis
c. Hyperaldosteronism
d. Panhyperpituitarism

e. Pyloric stenosis

A

a. Congenital adrenal hyperplasia

211
Q

BB
A 6-week-old breast-fed infant presents with a history of vomiting after feeds since
birth. He is growing along the 0.4th centile for weight. A pyloric mass is not palpable
during a test feed. The plasma chloride is 102 mmol/l (NR 96-110). What is the most
likely cause?
a. Cow’s milk protein allergy
b. Duodenal atresia
c. Gastro-esophageal reux
d. Malrotation
e. Pyloric stenosis

A

c. Gastro-esophageal reux

212
Q

BB
A 15-year-old basketball player complains of pain in his knees. Clinical examination
reveals, in addition to tenderness, a swollen and prominent tibial tubercle. X-rays of
the knee are unremarkable. What is the most likely diagnosis?

a. Gonococcal arthritis
b. Legg-Calve-Perthes disease
c. Osgood-Schlatter’s disease
d. Popliteal cyst
e. Slipped capital femoral epiphysis

A

c. Osgood-Schlatter’s disease

213
Q

BB
A 12-year-old boy has presented to A+E unable to weight-bear on his right leg due to
pain in the hip. He grazed his thigh 6 days ago whilst playing football, and for the
last 48 hours has had high fevers. Bloods taken today show a CRP of 240 and a WCC
of 24.5. A hip x-ray shows a periosteal reaction in the proximal femur but no fracture
or joint eusion.
The most likely diagnosis is:

a. Osteomyelitis
b. Perthe’s disease
c. Septic arthritis
d. Slipped upper femoral epiphysis
e. Transient synovitis

A

a. Osteomyelitis

214
Q

BB
A post-term infant is delivered by emergency Caesarian section for a moderate
placental abruption. On examination the infant’s oxygen saturation is 85% in air and
there is marked respiratory distress. A CXR reveals bilateral patchy inltrates.
What
is the most likely diagnosis?

a. Bacterial pneumonia
b. Bronchiolitis
c. Congenital heart disease
d. Meconium aspiration
e. Respiratory distress syndrome

A

d. Meconium aspiration

215
Q
BB
A 14 year old girl is stung by a bee and develops urticaria within 20 minutes of the
sting. What is the most likely immunological mediator?
a. Arachidonic acid
b. Eosinophils
c. IgE antibodies
d. Macrophages
e. T-cells
A

c. IgE antibodies

216
Q

BB
Two infants are born at 36 weeks’ gestation. One infant weighs 2600g at birth and an
unrelated second infant weighs 1600g. Which of the following conditions is the
second baby more likely to have?

a. Congenital malformations
b. Hyperglycaemia
c. Low haematocrit
d.
Occipito-frontal (head) circumference that is small compared to
body weight
e. Surfactant deciency

A

a. Congenital malformations - causing the low BW

217
Q

BB
A 2 year old presents with frank haematuria, some abdominal pain and rigors.
Which investigation is most likely to reveal the underlying diagnosis?

a. 24-hour urine protein quantication
b. Blood culture
c. Renal biopsy
d. Ultrasound scan of the abdomen
e. Urine M,C&S

A

e. Urine M,C&S

218
Q

BB
A 3-month-old breast-fed girl presents with a 3 day history of increasing
breathlessness and diculty
with feeding. On examination she is tachypnoeic and
hypoxic with no crepitations or wheeze and no abnormal upper airway signs. Her
mother declined antenatal blood tests. What is the most likely underlying cause of
this child’s respiratory illness?
a. Group B streptococcal infection
b. HIV infection
c. Inuenza
virus
d. Respiratory syncitial virus
e. Viral meningitis

A

b. HIV infection

219
Q

BB
A ten-year-old girl presents with recurrent abdominal pain. Of the following
symptoms, which is most suggestive of an organic aetiology?
Selected
Answer:

Answers: a. Periumbilical location
b. She has lost touch with her friends at school
c. She wakes from sleep with pain at night
d.
The pain has persisted intermittently for more than three
months
e. The pain is so severe that she cannot attend school

A

c. She wakes from sleep with pain at night

220
Q

BB
A term infant delivered by elective Caesarian section develops tachpnoea, grunting
and recession at 30 minutes of age and is hypoxic. A CXR shows flattened
diaphragms with fluid
in the right horizontal fissure
and well-aerated lung fields.

The infant improves after four hours and no longer requires oxygen. What is the
most likely diagnosis

Answers: a. Meconium aspiration

b. Patent ductus arteriosus
c. Primary pulmonary hypertension
d. Respiratory distress syndrome
e. Transient tachypnoea of the newborn

A

e. Transient tachypnoea of the newborn

221
Q

BB
A two year old boy is brought into A+E by ambulance having a generalised tonicclonic
seizure. Over the last 24 hours he has had a slightly runny nose, and his mum
has been giving him regular Paracetamol for a low grade fever. Mum also
comments that she herself had a cold sore recently. On arrival at A+E he is still
having the seizure. What would be the most appropriate immediate action?
Selected Answer: [None Given]
Answers: a. Do a lumbar puncture
b. Give buccal midazolam
c. Give IV acyclovir
d. Give IV phenytoin
e. Organise a CT head

A

b. Give buccal midazolam